Your Ad Here

Sunday, October 26, 2008

VIJAYA BANK PROBATIONARY OFFICERS EXAM.2008

(Held on 2-3-2008)Descriptive Language


Q.1. Heavy industries and labour intensive small indusries cannot exist together.Do you agree?comment as to how these two have co-existed in our country ?
Q.2. Poverty is a curse no doubt, but poverty is also not a cure for all the ills to ensure social welfare of the people.Comments.
Q.3. 'India's strength lies in its people.'Do you agree with this statement ?
Answer. Undoubtedly, India's strength lies in its people, through which,economic and social development ispossible.Therefore, the 'Labour policy'in india has evolved in response to specifice needs of the situationto suit requirments of planed economic development and social justice, so as to maintain industrial peaceand promoting the welfare of labour accordingly ,several acts have been framed like : the minimum Wages Act,1948;payment of wages Act,1936, Payment of bonus Act 1965;Industrial Disputes Act,1947, Maternity Benefit Act,1961,Employees state insurance Act 1948,Payment of gratuity Act 1972, etc. Through these framed acts, the people working in various sectors viz.Industries ,Education,Agro-Industries, Post Office/Railway Management etc.Get justice and remain happy.In fact,it is our strength.
India has -10 Bio-Geographic Zones; 14 Biosphere Reserves; 15 Agro-Climatic Zones; 20 Agro-Ecological Zones(AEZ)and 60 AESZ(AE Sub-Zones), where various types of vegetation/Forest;Crops Medicinal/Tea/Bamboo cultivation,Animals etc.Can easily be seen and so also ,India has 2 Bio-diversity Region,out of world 14 Bio-diversity Regions,situated at East-West Coasts and Ne regions, Which is a proud to us. The peopleresiding in these belts, in fact,this is our strength.
Q.4. What is microfinancing and how its progressing in our country ? Discuss.
Answer. Micro-financing means small amount of monetry finance, normally given for small sector sector of development i.e. small loans provided to 'poor to poor' for small sector improvement;which is given for many purposes; for example--small savings,loan credit,social safety like insurance etc.

The credit amount may vary from Rs. 5,000 to Rs. 25,000 of which return/back guarantee is 100% and the total payment is done with in 1 to 3 years. In this system, the credit is available as per people needs like KCC (Kisan Credit Card).
The best example of micro financing given to SHG’s (Self-Help Groups), having 10-12 participants and all get benefits of it, especially to establish small agro-based industries.
In fact, the poverty in our country is more, having a wide gap between capitalists and poor men. Therefore, micro-financing is in progress, with the men objective for providing this micro-finance to ‘poor to poor’, which is the need of the time and future also.
Q. 5 Discuss about the gender inequality in our country and its impact on our socio-economical development.
Ans. One of the important indices of population concentration is the density of population i.e. number of persons/km2, which was 324 per km2 in census 2001; being maximum 903 per km2-West Bengal, 881, per km2-Bihar (Second) and 819 per km2-Kerala (Third place).
Sex ratio i.e. number of females per 1000 males is an important social indicator to measure the extent of prevailing equality between males and females; which had always remained unfavourable to females in our country. As per the Census w.e.f. year 1901-2001, there appeared a decline in sex ratio, that can be observed from these Censuses-
Sex Ratio (1901-2001)

Census Year=============Sex Ratio (Females per 1,000 males)

1901=====================972

1911=====================964

1921=====================955

1931=====================950

1941=====================945

1951=====================946

1961=====================941

1971=====================930

1981=====================934

1991=====================926

2001=====================933

Women play a significant role in agriculture in the country, which has a direct relationship with our socioeconomic development. Since, women have a vital role in agriculture-especially in the field of Horticulture- Floriculture (flower cultivation), Pomology (fruit cultivation) and Olericulture (vegetable production), paddy transplanting, weeding/interculture in crops, harvesting, and Animal Husbandry and Dairy Sector, especially the SC/ST and Non SC women etc. NRC-WA (National Research Centre for Women in Agriculture-ICAR) Bhubaneshwar (Orissa) is doing an excellent work for empowerment of women in agricultural research and technology transfer and found that 43.7% of women’s time goes to agriculture. If the sex ratio goes down. It will surely adversely affect the socio-economic development of the society.

BIHAR PUBLIC SERVICE COMMISION (Pre.) EXAM.2008

GENERAL STUDIES (held on 25-5-2008)

Q.1. What is sequence number of Ms. Pratibha patil as President of the Republic of India ?
(A) 10th (B) 11th (C) 12th (D) 13th

Q.2. Abill presented in parliament becomes an act after--
(A) It is passed by Both the Housees
(B) The president has given his assent
(C) The prime Minister has Signed it
(D) The supreme Court has declared it to be within the competence of the Union-Parliament

Q.3. According to our constitution the Rajya-sabha--
(A) Is desolved once in two years (B) In desolved every five years. (C) Is dissolved every six years (D) Is not subject of dissolution

Q.4. A High Court Judge Addresses his letter of resignation to--
(A) The President (B) The Chief Justice of India (C) The Chief Justice of High court (D) the Governor of the state

Q.5. As a non-member who can Participatein the proceedings of either House of parliament--
(A) Vice-President(B) Chief Justice (C) Attorny-General (D) Chief Election Commisioner

Q.6. Fundamental Duties were incorporated in the Consititution of India by the--(A) 32nd Amendment Act (B) 42nd Amendment Act (C) 15th Amendment Act (D) 46th Amendment Act

Q.7. How many Languages are recognised as regional language in the constittution ?
(A) 12 (B) 13 (C) 14 (D) 22

Q.8. Consider the following the words:
1.Socialist 2. Democratic 3. Sovereign 4.secular

Q.9. In which house is the presiding officer not a member of that house ?
(A) Lok-Sabha (B) Rajya-sabha (C) Vidhan-sabha (D) Vidhan-Parishad

Q.10.By which constitutional amendment bill was the voting age redused from 21 years to 18 years ?
(A) 48th (B) 57th (C) 61st (D) 63rd


ANSWER-- 1.(C) 2.(B) 3.(D) 4.(A) 5.(C) 6.(B) 7.(D) 8.(A) 9.(B) 10.(C).



Q.11. If the position of President and Vice-president are vacant, who officiates at the president of india ? (A) The Prime Minister
(B) The Chief Justice of India
(C) The Speaker of Lok-Sabha
(D) None of these

Q.12. A panchayat samiti at the block level is--
(A) An advisory body
(B) An Administrative authority
(C) A consultant Committee
(D) A supervisory authority

Q.13. Acording to the world bank's leatast development report, the position of the indian economy in the world is the ---
(A) Largest
(B) Smallest
(C) Second Largest
(D) Fifth Largest

Q.14. If the cash reserve ratio is lowered by the RBI, it's impact oncredt creation will be--
(A) Increase it
(B) Decrease it
(C) No impact
(D) None of the other

Q.15. Indian Development Forum (IDF) was earlier Known as ---
(A) Aid India Consortium
(B) Aid India Bank
(C) World Bank
(D) None of these

Q.16. It will be true to classify India as--
(A) A food-deficit economy
(B) A labour-surplus economy
(C) A trade surpluse economy
(D) A capital surplus economy

Q.17. Modvat is related to--
(A) sales Tax (B) Wealth Tax (C) Income Tax (D) Excise Duty

Q.18. The eleventh plan's objective is--
(A) Removal of poverty
(B) Inclusive growth
(C) Growth with social-justice
(D) Development of minorities

Q.19. Rainbow revolution is related with--
(A) Green-revolution (B) White-revolution (C) Blue-revolution (D) All the above

Q.20. The one rupee note bears the signature of the--
(A) Secretary, Ministry of finance
(B) Governor,RBI
(C) Finance Minister
(D) None of these


Answer : 11. (B) 12.(B) 13.(D) 14. (A) 15.(A) 16.(B) 17.(D) 18.(B) 19.(D) 20.(A).

State Bank of India Probationary Officer 2008 :Solved Paper

General Awareness & Computer Knowledge (Held on 27-07-2008)

Q.1. Which of the following statement(s) reflects India's stand on new World Trade Organization's proposal on the issue of the subsidy to agro products ?
(1) India would protect the interest of its poor farmers who can not bear the burden of the cost of the products if they have to sell them on cheaper rate.
(2) Indian industry needs more flexibility as compared to the industry in developed countries.
(3) WTO has set up a new committee under the chairmanship of Putin of Russia to look into the issue of subsidy and find out a solution to the problem acceptable to all the parties.
(A) Only 1
(B) Only 2
(C) Only 3
(D) Both 1 & 2
(E) None of these
Ans : (A)

2. Which of the following statement(s) is / are true about the 11th five year plan of India ?
(1) The highest priority is given to exports ,education and development of steel sector.
(2) The plan aims at achieving the growth at 10 % level by the end of the plan.
(3) The Rashtriya Krishi Vikas Yojana is revived in such a way so that it can be achieve a growth nof 4 % for agriculture sector.
(A) Only 1
(B) Only 2
(C) Only 3
(D) Both 1 & 2
(E) None of these
Ans : (D)

3. As per the figures released recently the Stock markets in emerging economies made handsome returns last year (2007-08). Which of the following stock markets amongst the BRIC nations registered highest increased (97 %) compared to its performance in 2006-07 ?
(A) China
(B) Brazil
(C) Russia
(D) India
(E) None of these
Ans : (A)

4. Bayana which was in news recently is a place in—
(A) Uttar Pradseh
(B) Madhya Pradseh
(C) Bihar
(D) Haryana
(E) Rajasthan
Ans : (E)

5. Which of the following statement (s) is / are correct about the Indo-Russia Bilateral Defence Ties ?
(1) Both the countries have decided to produce a fifth generation fighter aircraft and a multi role cargo plane.
(2) India has agreed to launch Russian missiles from its base at Chandipur incase some other nation attacks on Russia
(3) India and Russia were able to sort out differences over the utilization of Rupee debt fund which is of Rs. 8000 crore at present.
(A) Only 1
(B) Only 2
(C) Only 3
(D) Both 1 & 3 only
(E) None of these
Ans : (A)

6. India has recently decided to put a wire fence on the 76 km. long border in
Dhudri district in Assam. The fence will separate India from—
(A) Nepal
(B) China
(C) Bangladesh
(D) Myanmar
(E) Pakistan
Ans : (C)

7. As we all know the Govt. of India is very keen about providing banking service to the people in rural and remote areas . However , banks feel that the cost of providing services in these areas for financial inclusion is quite high if services are offered through the "traditional branch channel ". What is / are the other alternative cost effective channel(s) to fulfill this aim of the Govt. of India ?
(1) Putting biometric ATMs.
(2) Introduce mobile banking.
(3) Issue credit or debit cards
(A) Only 1
(B) Only 2
(C) Only 3
(D) All 1, 2 & 3
(E) None of these
Ans : (B)

8. The Govt. of India recently issued a notification altering constituencies in 24 States and Union territories. This is done as per the provision of which of the following Acts?
(A) Delimitation Act
(B) Census Act
(C) Parliament Act
(D) State Reorganization Act
(E) None of these
Ans : (A)

9. As we read very frequently in the newspaper fraudulent activities in financial world is still continuing despite various efforts made by various countries to prevent them. As an estimate by IMF, what is the probable amount derived from such illegal activities which is laundered through the world's formal financial systems every year? About -
(A) US$ 500 billion
(B) US$ 800 billion
(C) US$ 1000 billion
(D) US$ 1250 billion
(E) US$ 1500 billion
Ans : (A)

10. Indian Prime Minister during his visit the Arunachal Pradseh a few months back asserted that the State is India's Land of Rising Sun. Which of the following countries did not like his statement and had raised some objection over his visit to Arunachal Pradseh which is very much a part of India?
(A) Myanmar
(B) China
(C) Bangladseh
(D) Bhutan
(E) Nepal
Ans : (B)

11. Which of the following is the amount of insurance cover provided to the workers of the unorganized sector under Rashtriya Swasthya Bima Yojana?
(A) Rs. 10,000/-
(B) Rs. 20,000/-
(C) Rs. 30,000/-
(D) Rs. 40,000/-
(E) Rs. 50,000/-
Ans : (C)

12. Which of the following is NOT one of the highlights Economic Survey presented in the Lok Sabha for the year 2007-08?
(A) Economy has "moved to higher growth phase"
(B) A trend of acceleration in domestic investment and saving rate is recorded.
(C) Buoyant growth in Govt. revenue.
(D) Indian Economy at market rate of exchange crossed 1 trillion US$ in current fiscal.
(E) Net Foreign direct Investment increased by 350% .
Ans : (E)

13. Which of the following is / are correct about sub prime crisis which was in news recently?
(1) It has reduced the flow of capital coming of Indian Stock Markets.
(2) The sub prime crisis is the major financial crisis whose origin is in the Russia and Brazil.It has adversely affected the economy of Europe and USA.
(3) The crisis started when some people started trading with false Share certificates of major blue chip US companies in the stock markets.
(A) Only 1
(B) Only 2
(C) Only 3
(D) Both 1 & 2
(E) None of these
Ans : (D)

14. The Prime Minister Gram Sadak Yojana (PMGSY) has a target to connect all habitations of which of the following levels with all weather roads by 2008-09?
(A) 1000+
(B) 2000+
(C) 3000+
(D) 4000+
(E) None of these
Ans : (A)

15. As we all know the value of the Indian rupee has appreciated in the recent past . The value of the Rupee is direct dependent on which of the following?
(A) Availability of commodities in the market.
(B) VAT introduced by the Govt. on various commodities.
(C) Size of total Forex Reserve of India currently.
(D) Capital flow in the market during a given period of time.
(E) None of these
Ans : (A)

16. Which of the following banks has taken over the Centurion Bank of Punjab?
(A) ICICI Bank
(B) IDBI Bank
(C) HDFC Bank
(D) AXIS Bank
(E) None of these
Ans : (B)

17. As per provision made in Bharat Nirman Programme the pace of creation of irrigation facilities is required to be raised from present level of 1.42 million hectare per year in such a way so that it can achieve the target level fixed for the same?
(A) 1.50 million hectares / year
(B) 1.75 million hectares / year
(C) 2.00 million hectares / year
(D) 2.25 million hectares / year
(E) 2.50 million hectares / year
Ans : (D)

18. Inflation has become major area of concern in India these days. What measures do the Govt. of India / RBI normally take to control the same?
(1) Fixation of Maximum Price of the Commodities
(2) System of Dual prices.
(3) Increased in supply of food grains.
(4) Control on credit and liquidity in market.
(A) Only 1
(B) Only 2
(C) Only 3
(D) Only 2, 3 & 4
(E) All 1, 2, 3 & 4
Ans : (D)

19. How much Education Cess is being levied on all the taxes to support elementary education for the children of age group of 6-14 years under Sarva Shiksha Abhiyan (SSA)?
(A) 0.50%
(B) 0.75%
(C) 1.00%
(D) 1.50%
(E) 2.00%
Ans : (E)

20. Which of the following is / are correct about various concessions given in the Union Budget for 2008-09?
(1) Five years tax holidays for setting up hospitals in Tier II & Tier III regions for providing health care in rural areas.
(2) Commodities tax transaction to be introduced.
(3) Banking cash transaction tax withdrawn.
(A) Only 1
(B) Only 2
(C) Only 3
(D) Only 2 & 3
(E) All 1, 2 & 3
Ans : (E)

21. Which of the following statements reflects the aim(s) Jawaharlal Nehru National Urban Renewal Mission (JNNURM)?
(1) Give focused attention to integrated development of infrastructure in selected cities.
(2) Provide basic services to the urban poors
(3) Scale up civic amenities and provision of utilities in urban areas.
(A) Only 1
(B) Only 2
(C) Only 3
(D) All 1, 2 & 3
(E) None of these
Ans : (D)

22. The Press Freedom Day is observed on which of the following days?
(A) 1st May
(B) 1st June
(C) 3rd May
(D) 3rd June
(E) 10th June
Ans : (C)

23. As per the decision taken by the Govt. of India now the National Rural Employment Guarantee Act is extended to all the districts of India. This means : it will now be applicable to about—
(A) 200 districts
(B) 300 districts
(C) 400 districts
(D) 500 districts
(E) 600 districts
Ans : (E)

24. As per the reports published by the Ministry of Finance recently the overseas borrowings by the Corporates and currency fluctuation has gone up by which of the following percentage points? About—
(A) 1%
(B) 2%
(C) 3%
(D) 4%
(E) 5%
Ans : (C)

25. The Govt. of India and Reserve Bank of India is planning to set up an autonomous "ATM corporation of India" in which various banks can get together for common ATM operations. If this happens how will we all be benefited by the same?
(1) It will be a cost effective measure as each bank can share its operational cost with others.
(2) Clients can withdrawn money from any ATM from any location. They are not required to go to their banks' ATM only.
(3) It will indirectly help the IT business also as India has a large pool of IT skilled manpower waiting for suitable placement.
(A) Only 1
(B) Only 2
(C) Both 1 & 2
(D) All 1, 2 & 3
(E) None of these
Ans : (C)

26. As per the news published in various newspapers the Govt. of India has decided to create a Strategic Reserve of—
(A) Oil & Petroleum
(B) Rasoi Gas
(C) Food grains
(D) Cement & Steel
(E) None of these
Ans : (C)

27. The Govt. of India had constituted a high level committee on financial sector reforms with Raghuram G.Rajan as its Chairman. Which of the following is / are the recommendations of the committee?
(1) Liberalize the interest rate that banks/institution can charge to ensure credit for poors.
(2) Create an office of the Ombudsman exclusively for financial sector.
(3) Do not allow foreign investors in domestic stock markets.
(A) Only 1
(B) Only 2
(C) Only 3
(D) Both 1 & 2
(E) None of these
Ans : (E)

28. Yves Leterme has taken over as the prime minister of—
(A) Newzealand
(B) Belgium
(C) Croatia
(D) Portugal
(E) None of these
Ans : (B)

29. The State Bank of India recently opened its 10000th branch in Puduvayal (Tamil Nadu). Only a bank in China has more branches than State Bank of India in the world. What is the name of that Chinese bank?
(A) Shanghai Commercial Bank
(B) Bank of China
(C) China Banking Corporation
(D) Industrial and Commercial Bank of China
(E) None of these
Ans : (D)

30. Many a times we read about 'Globalization' and its impact on business practice in India. Which of the following is NOT one of the important parameters of 'Globalization'?
(1) Reduction of trade barriers to permit free flow of goods and services amongst various nations.
(2) Developing an environment in which free flow of capital can take place.
(3) Promoting local industry to cater to the needs of the consumers in Tier II and Tier III cities. The production from metros should be reserved only for exports.
(A) Only 1
(B) Only 2
(C) Only 3
(D) All 1,2 & 3
(E) Both 2 & 3
Ans : (D)

31. Many times we read a term CBS used in banking operation. What is the full form of the letter 'C' in the term 'CBS'?
(A) Core
(B) Credit
(C) Continuous
(D) Complete
(E) None of these
Ans : (A)

32. As per news published in newspapers the Oil & Natural Gas Commission (ONGC) has purchased some oil assets in Latin American Countries along with one other major company of India . What is the name of that company?
(A) Hindustan Petroleum Corporation
(B) Indian Oil Corporation
(C) Reliance Industries Ltd.
(D) National Mineral Development Corporation of India
(E) None of these
Ans : (E)

33. India's First centre to monitor climate change was opened recently at—
(A) New Delhi
(B) Chennai
(C) Jaipur
(D) Bangalore
(E) Kolkata
Ans : (B)

34. Which of the following Acts was framed specially to deal more effectively with the problem of Non-Performing Assets in banking system?
(A) SARFAESI ACT.
(B) Banking Regulation Act.
(C) Foreign Exchange Management Act.
(D) Industrial Dispute Act.
(E) None of these
Ans : (A)

35. The————shows all the websites and pages that you have visited over a period of time.
(A) status bar
(B) task bar
(C) history list
(D) tool bar
(E) None of these
Ans : (C)

36. A proxy server is used for the which of the following?
(A) To provide security against unauthorized users
(B) To process client requests for web pages
(C) To process client requests for database access
(D) To provide TCP/IP
(E) None of these
Ans : (A)

37. A repair for a known software bug,usually available at no charge on the internet, is called a(n)—
(A) version
(B) Patch
(C) Tutorial
(D) FAQ
(E) None of these
Ans : (B)

38. In the URL http://www.prenhall.com, the portion lebelled http is the—
(A) host
(B) domain name
(C) protocol
(D) top-level domain
(E) None of these
Ans : (C)

39. Which of the following organizations has recently given a US$ 600 million loan to Power Grid Corporation of India,a Govt. run organization?
(A) Asian Development Bank
(B) World Bank
(C) International Monetary Fund
(D) Bank of America
(E) None of these
Ans : (B)

40. The ability to easily add additional users meAns : that a network is—
(A) Scalable
(B) dedicated
(C) Decentralized
(D) Secure
(E) None of these
Ans : (A)

41. In a database, a field is a—
(A) lable
(B) table of information
(C) group of related records
(D) category of information
(E) None of these
Ans : (C)

42. The number of pixels displayed on a screen is known as the screen—
(A) resolution
(B) colour depth
(C) refresh rate
(D) viewing size
(E) None of these
Ans : (A)

43. Wor processing , spreadsheet , and phoyo-editing are example of—
(A) application software
(B) system software
(C) operating system software
(D) platform software
(E) None of these
Ans : (A)

44. If you wish to extend the length of the network without having the signal degrade, you would use a—
(A) repeater
(B) router
(C) gateway
(D) switch
(E) None of these
Ans : (A)

45. A————is approximately a million bytes.
(A) gigabyte
(B) kilobyte
(C) megabyte
(D) terabyte
(E) None of these
Ans : (C)

46. The time it takes a device to locate data and instructions and make them available to CPU is known as—
(A) clock speed
(B) a processing cycle
(C) CPU speed
(D) access time
(E) None of these
Ans : (D)

47. ——————controls the way in which the computer system functions and provides a means : by which users are interact with the computer.
(A) The platform
(B) The operating system
(C) Application software
(D) The motherboard
(E) None of these
Ans : (B)

48. ——————meAns : that the data contained in a database is accurate and reliable.
(A) Data redundancy
(B) Data integrity
(C) Data reliability
(D) Data consistency
(E) None of these
Ans : (B)

49. A(n) —————— is aprivate corporate network,used exclusively by company employees.
(A) Internet
(B) local area network
(C) peer-to-peer
(D) intranet
(E) None of these
Ans : (B)

50. A characteristic of a file server is which of the following?
(A) Manages file operations and is shared on a network
(B) Manages file operations and is limited to one PC
(C) Acts as fat client and is shared on a network
(D) Acts as fat client and is limited to one PC
(E) None of these
Ans : (A)

R.A.S/R.T.S (Pre.) Examination,2007

Public Administration : Solved Paper(Held on 23-12-2007)

Q.1. Under which Article of Constitution the Governor of State is bound to lay before House of Legislature the Annual Financial Statement of State ?
(A) Article-111
(B) Article-117
(C) Article-202
(D) Article-203
Ans : (C)

2. In Britain, responsibility for Central Co-ordination and Management of Civil Services is divided between—
(A) Cabinet office and Treasury
(B) Treasury and Home Department
(C) Prime Minister and Home Department
(D) Cabinet Office and Prime Minister Office
Ans : (B)

3. Who is in-charge of Office of Public Service and Science in Britain ?
(A) Cabinet Secretary
(B) The Chancellor of the Duchy of Lancaster
(C) Lord Chancellor
(D) Chancellor of Exchequer
Ans : (A)

4. The term ‘Economic and Social Planning’ is included in Indian Constitution. In which part of following this provision has been made ?
(A) Directive Principles for State Policy
(B) Preamble of Constitution
(C) Union List
(D) Concurrent List
Ans : (D)

5. Who was the first Lokayukta of Rajasthan ?
(A) K.P.S. Menon
(B) I.G. Dua
(C) D.P. Gupta
(D) M.C. Jain
Ans : (C)

6. Match list-I with list-II and select the correct answer—
List-I (Kind of Budget)
(a) Line Item Budget
(b) Performance Budget
(c) Planning Programming Budget.
(d) Zero-based Budget.
List-II (Basic Orientation of Budget)
1. Control
2. Decision making
3. Management
4. Planning
Codes :
(a) (b) (c) (d)
(A) 1 3 4 2
(B) 1 3 2 4
(C) 1 2 3 4
(D) 1 4 2 3
Ans : (A)

7. Personnel administration of employees of Zila Parishad and Panchayat Samiti in Rajasthan is managed by—
(A) Zila Parishad
(B) State Government
(C) District Administration
(D) Zila Establishment Committee
Ans : (A)

8. Who will make rules for the more convenient transaction of business of Government of India and for allocation among Ministers of the said business ?
(A) President
(B) Cabinet Secretariat
(C) Prime Minister
(D) Parliament
Ans : (A)

9. Which one is not characteristics of ‘Sala Model’ of Riggs ?
(A) Actual behaviour is similar to formal expectations
(B) Institutional corruption
(C) Nepotism in recruitment
(D) Above all
Ans : (A)

10. Discussion on organisation of Planning Commission is incomplete without reference to its attached bodies. Consider the following—
1. Programme Evaluation Organisation
2. National Informatic Centre
3. Plan Finance Division
4. Department of Rural Development
Which organisations are attached to Planning Commission ?
Codes :
(A) 1 and 2
(B) 1 and 3
(C) 2 and 3
(D) 1 and 4
Ans : (A)

11. In which year the institution of Lokayukta was established in Rajasthan ?
(A) 1970
(B) 1971
(C) 1972
(D) 1973
Ans : (D)

12. Who had recommended for productive planning with a constitutional status in India ?
(A) Pt. Jawaharlal, Nehru
(B) Dr. Rajendra Prasad
(C) Dr. B. R. Ambedkar
(D) K.C. Neogi
Ans : (C)

13. ‘The Collector sometimes when overburdened with work, grumbles that he has the life of a dog, but knows that he has work of a man.’ Who said it ?
(A) L.S.S.O. Malley
(B) Ramsay MacDonald
(C) Mangat Rai
(D) P.R. Dubhasi
Ans : (A)

14. The CLECTS is prismatic society of Riggs are—
(A) Elite groups who use traditional clouts to win election
(B) Hereditary leaders of castegroup
(C) Elected leader of middle class
(D) Influential political leader
Ans : (A)

15. The Parliamentary Commissioner for Administration in England is Parliamentary Ombudsman.
Consider the following—
1. Is independent of Government.
2. Investigates complaints referred through Member of Parliament.
3. Investigates complaint about Government policy.
4. Reports his findings to Member of Parliament.
What are the characteristics of Parliamentary Commissioner for Administration ?
Codes :
(A) 1 and 2
(B) 1, 2 and 3
(C) 1, 2 and 4
(D) Above all
Ans : (C)

16. Who has coined the term Cybernetics ?
(A) Weaver
(B) Wilber Schramm
(C) Claude Shannon
(D) Norbert Weiner
Ans : (D)

17. Consider the following steps—
1. Identification of problem
2. Search for alternatives
3. Evaluation of alternatives
4. Comparison of alternatives
Which of the above is the correct step of decision-making ?
(A) 1, 4, 3, 2
(B) 1, 3, 2, 4
(C) 1, 2, 4, 3
(D) 1, 2, 3, 4
Ans : (C)

18. According to Likert, which of the following types of leadership is the most successful and effective ?
(A) Exploitative-authoritative
(B) Benevolent-authoritative
(C) Participative
(D) Consultative
Ans : (C)

19. The Theory X assumption did not cover which of the following ?
(A) Most of workers not inherently dislike work
(B) Most of people must be coerced, controlled or threatened for the achievement of organizational goals
(C) The average human being prefers to be directed
(D) They have relatively little ambitions and wants
Ans : (A)

20. Which one of the following is covered under Eleventh Schedule of the Indian Constitution ?
(A) Union List
(B) State List
(C) Allocation of seats in the Council of States
(D) Jurisdiction of functions of Panchayats
Ans : (D)

21. In a communication process, the sender of information seeks to establish a commonness with the receiver of information.
Consider the following—
1. When interacting with others, words, letters etc. are used.
2. Two or more persons are involved in communication.
3. The interaction process is motivated by sharing of information towards common
ends.
4. Above all.
What are the characteristics of communication ?
Codes :
(A) 1
(B) 4
(C) 3
(D) 2
Ans : (B)

22. Which of the following Provisions of Indian Constitution provides constitutional safeguards to Civil Servants ?
(A) Article 309
(B) Article 310
(C) Article 311
(D) Article 32
Ans : (C)

23. Before the introduction of money bill in Lok Sabha, whose recommendation is necessary ?
(A) President
(B) Prime-Minister
(C) Speaker
(D) Chairman of the Rajya Sabha
Ans : (A)

24. The reservation in appointments in favour of any backward class has a constitutional base. In which Article of Indian Constitution such provision has been made ?
(A) Article 16(i)
(B) Article 16(ii)
(C) Article 16(iii)
(D) Article 16(iv)
Ans : (D)

25. Match list-I with list-II and select the correct answer—
List-I (Statements)
(a) Budget is series of goals with price tag attached.
(b) Finance is the fuel of Administrative Machinery.
(c) Government is Finance.
(d) All undertakings depend upon finance, hence foremost, attention shall be paid to the treasury.
List-II (Thinkers)
1. Kautilya
2. Lloyd George
3. P. K. Wattal
4. Aron Wildavski
Codes :
(a) (b) (c) (d)
(A) 4 2 1 3
(B) 1 2 3 4
(C) 3 1 4 2
(D) 4 3 2 1
Ans : (D)

26. Match list-I with list-II and select the correct answer —
List-I (Thinkers)
(a) Mouney and Reiley
(b) Etzioni
(c) Marvick
(d) Gouldner
List-II (Kinds of Leadership)
1. Institutionists, Specials, Hybrids.
2. Titular Leaders, controller, true organisers.
3. Officials, Informal Leaders, Formal Leaders.
4. Locals and Cosmopolitans.
Codes :
(a) (b) (c) (d)
(A) 2 3 1 4
(B) 1 3 2 4
(C) 1 2 4 3
(D) 2 1 4 3
Ans : (B)

27. Which one of the following is not the merit of Rank classification ?
(A) It is easy to understand and administer
(B) It defines the contents of any job in detail
(C) It encourages career opportunities
(D) It attracts competent persons to the service
Ans : (B)

28. Match list-I with list-II and select the correct—
List-I (Item)
(a) Constitution of Municipalities
(b) Reservation of seats in Municipal bodies.
(c) State Finance Commission.
(d) Term of Municipalities.
List-II (Articles of the Constitution)
1. 343 Q
2. 243 T
3. 243 Y
4. 243 U
Codes :
(a) (b) (c) (d)
(A) 1 2 4 3
(B) 1 2 3 4
(C) 3 2 1 4
(D) 1 4 3 2
Ans : (B)

29. Which one of the following Writs gives the direction to an official for performance of duty ?
(A) Habeas Corpus
(B) Mandamus
(C) Certiorari
(D) Quo-Warranto
Ans : (B)

30. Which of the following are the tools of budgetary control exercised by the Parliament ?
1. Cut Motions
2. Financial Committees of Parliament.
3. Comptroller and Auditor General of India.
4. Approval of estimates for demands.
Codes :
(A) 1 and 2
(B) 1 and 3
(C) 2 and 3
(D) Above all
Ans : (D)

31. Which one of the following is not the function of Public Officials ?
(A) To implement Government Policy
(B) To advise the political executive on policy matters
(C) To maintain Public relations
(D) To make the decision on policy
Ans : (D)

32. Which one of the following statements is not correct in respect of French Civil Services ?
(A) Recruitment and training are combined in the ENA
(B) The corps is a basic unit of Civil Services
(C) Ecole Polytechnique does not conduct training for higher technical corps
(D) It is elitist and very legalistic
Ans : (C)

33. ‘No Tax shall be levied or collected except by authority of Law’. Which Article of Indian Constitution provides this ?
(A) 110
(B) 117
(C) 265
(D) 266
Ans : (C)

34. The elections for various Panchayati Raj bodies in Rajasthan are managed by—
(A) Election Commission
(B) State Election Commission
(C) Directorate of Panchayati Raj
(D) Zila Parishad
Ans : (B)

35. The body of apex of the system of Administrative Court in France is the—
(A) Constitutional Council
(B) Economic and Social Council
(C) Council of State
(D) Court of Accounts
Ans : (C)

36. Who among the following is considered as first comparativist in Public Administration ?
(A) Woodrow Wilson
(B) Goodnow
(C) L.D. White
(D) Fred Riggs
Ans : (A)

37. Which of the following is not characteristic of Development Administration ?
(A) Goal orientation
(B) Change orientation
(C) Result orientation
(D) Effectiveness
Ans : (C)

38. Which one of the following is not correct about anonymity of Civil Servants ?
(A) They cannot be criticised by name in the Legislature
(B) The minister concerned defends actions of Civil servant of his department
(C) Civil servants perform their official duty as per rules
(D) They cannot be held responsible for their acts of omission and commission
Ans : (D)

39. In Rajasthan, State PIanning Board was created on the recommendation of—
(A) Rajasthan Administrative Reforms Committee, 1967
(B) Secretariat Procedure Committee, 1971
(C) Secretariat Re-organisation Committee, 1969
(D) Secretaries Committee of the Government of Rajasthan, 1962
Ans : (A)

40. ‘The Chief Secretary’s job is not a technician’s or even a professional’s, he is not a knowledgable engineer, nor even a first class magistrate, he is a part of process of Government and in a democratic republic, part of human process.’ Who said it ?
(A) E.N. Magat Rai
(B) Dharmvir
(C) V. D. Murti
(D) Mohan Mukherjee
Ans : (A)

41. ‘Heuristics’ in context of decision making imply—
(A) Error and Trial method
(B) Rational decision making
(C) Decision based on deep analysis
(D) Decision based on bounded rationality
Ans : (A)

42. The ‘quorum’ for meetings of any Panchayati Raj body as per Rajasthan Panchayati Raj Act is—
(A) 1/10 of total members
(B) 1/3 of total members
(C) 1/4 of total members
(D) None of above
Ans : (B)

43. Consider the following statements—
1. Budget is an economic horoscope of the country
2. Budget is a political document which provides a glimpse of entire philosophy of Government
3. Budget is formulated by Legislature
4. The budget is the nut-bolts of Public Policy
Which of the above are characteristics of Budget ?
(A) 1 and 2
(B) 1, 2 and 4
(C) 1, 2 and 3
(D) 2, 3 and 4
Ans : (B)

44. Match list-I with list-II and select the correct answer—
List-I (Model)
(a) Linking Pin Model
(b) Decision Tree
(c) Garbage Cane Model
(d) Theory X and Theory Y
List-II (Given by)
1. Mc Gregor
2. Likert
3. Duncan
4. Cohen, March and Olson
5. Fred Fiedler
Codes :
(a) (b) (c) (d)
(A) 2 3 4 1
(B) 3 4 5 1
(C) 4 5 3 1
(D) 2 4 3 1
Ans : (D)

Directions—(Q. 45-46) The following two items consists of two statements, one labelled as the ‘Assertion’ (A) and the other as ‘Reason’ (R). Carefully examine these two statements and select the answers to these items using codes given below—
Codes :
(A) Both (A) and (R) are true and (R) is the correct explanation of (A).
(B) Both (A) and (R) are true, but (R) is not correct explanation of (A).
(C) (A) is true, but (R) is false.
(D) (A) is false, but (R) is true.
Ans : (B)

45. Assertion (A) : The Council of Ministers shall be collective responsible to the House of People.
Reason (R) : The Minister shall hold office during the pleasure of President.
Ans : (B)

46. Assertion (A) : Ordinances should be promulgated only where absolutely necessary. A responsible Government should be careful about it.
Reason (R) : It gives the executive a clear power to legislate and is an encroachment on the jurisdiction of the Parliament.
Ans : (C)

47. Who said that in the science of administration whether public or private, the basic good is efficiency ?
(A) Urwick
(B) Luther Gulick
(C) Henri Fayol
(D) J.D. Mooney
Ans : (C)

48. Match list-I with list-II and select the correct answer—
List-I (Statements)
(a) Public Administration stand in danger of … senescence.
(b) That Public Administration is that lusty young giant of a decade ago, may now evaporate as a field.
(c) Public Administrationists as administrative Platonists.
(d) Public Administrationists has an uncomfortable and second class citizenship.
List-II (Authors)
1. Martin Landau
2. Paul Appleby
3. Waldo
4. F.M. Marx
Codes :
(a) (b) (c) (d)
(A) 4 3 2 1
(B) 4 2 1 3
(C) 1 2 3 4
(D) 4 1 2 3
Ans : (D)

49. Who said that Politics has to do with policies or expression of State will while Administration has to do with the execution of these policies ?
(A) Woodrow Wilson
(B) Goodnow
(C) Waldo
(D) J.M. Gaus
Ans : (A)

50. Who said that administration is knowledge, knowledge is power. Administration is power, this simplistic syllogism is major reality of our post-industrial age—
(A) James D. Carrol
(B) James Fallows
(C) Jimmy Carter
(D) John F. Kennedy
Ans : (A)

51. Organisation is the form of every human association for attainment of a common purpose. Who has given this definition ?
(A) Gulick
(B) James D. Mooney
(C) L.D. White
(D) Simon
Ans : (B)

52. Willy-nilly, administration is every one’s concern. If we wish to survive, we had better be intelligent about it. Who said it ?
(A) Henri Fayol
(B) Adams Brooks
(C) Dwight Waldo
(D) Wallace B. Donham
Ans : (B)

53. While making comparison with private administration, who said that Public Administration is adrift with compass or anchor ?
(A) Simon
(B) Peter Self
(C) Paul Appleby
(D) Ludwig Von Mises
Ans : (D)

54. Which one is not characteristics of closed model of organisation ?
(A) Routine task
(B) Conflict within organization is adjusted by interaction with peers
(C) Task specialisation
(D) Means are emphasized
Ans : (B)

55. Which one of the following is not a driving paradigm of New Public Management ?
(A) Devolving authority
(B) Developing competition and choice
(C) Providing responsive service
(D) Emphasis on procedure
Ans : (D)

56. Public Management is merger of normative orientation of Traditional Public Administration and the instrumental orientation of general management. Who said it ?
(A) L.A. Gunn
(B) Perry and Kramer
(C) Christopher Hood
(D) Tullock
Ans : (B)

57. Who was the first present give red-tape its sociological baptism ?
(A) Lord Lytton
(B) Herbert Spencer
(C) Sidney Smith
(D) Alexander Leighton
Ans : (A)

58. Who said that organisation is the machine of management in its achievement of the ends determined by administration ?
(A) William Schulze
(B) Oliver Sheldon
(C) Gladden
(D) Luthur Gulick
Ans : (A)

59. Which of the following is concerned with the modification of role expectancies and welding of divergent elements to preserve organisation integrity ?
(A) Galbraith’s Information Processing
(B) Bakke’s Fusion Process
(C) Duncan’s Decision Making Process
(D) Likert’s Group Theory
Ans : (B)

60. The older theory or organization is not science, as often claimed, but theology in sense that it is a set of beliefs accepted on faith, not on demonstration of empirical reality. Who has given this statement ?
(A) F.W. Taylor
(B) Herbert Simon
(C) Earl Latham
(D) Elton Mayo
Ans : (B)

61. Hierarchy is the linchpin that locks the form. Who said it ?
(A) Allen Schick
(B) Earl Latham
(C) Golembiewski
(D) Stephen K. Bailey
Ans : (A)

62. Match list-I with list-II and select the correct answer—
List-I (Organisation)
(a) Project Organisation
(b) Matrix Organisation
(c) Horizontal Organisation
(d) Network Organisation
List-II (Characteristics)
1. Existence of horizontal and diagonal relationship.
2. Facilitate co-operation, team work and customer orientation.
3. Independent Unit have a link with a web of other unit.
4. Project organisation plus a functional organisation.
5. Flexible to facilitate management change.
Codes :
(a) (b) (c) (d)
(A) 1 4 2 3
(B) 1 5 2 3
(C) 5 2 1 3
(D) 2 4 5 3
Ans : (C)

Directions—(Q. 63-72) The following 10 items consist of two statements, one labelled as the ‘Assertion’ (A) and the other as ‘Reason’ (R). You are to examine these two state-ments carefully and select the answers to these items using the codes given below—
Codes :
(A) Both (A) and (R) are true and (R) is the correct explanation of (A).
(B) Both (A) and (R) are true, but (R) is not correct explanation of (A).
(C) (A) is true, but (R) is false.
(D) (A) is false, but (R) is true.

63. Assertion (A) : The Principle of Organisation is from the literature of Business Administration.
Reason (R) : There is larger area of overlapping interest and ideas between two fields, however, and in the matter of organizational analysis, Public Administration is heavily indebted of Business Administration.
Ans : (A)

64. Assertion (A) : Administration is a fine art.
Reason (R) : It summons an imposing body of special talents on behalf of a collaborative creation which is integral to the conduct of civilized living today.
Ans : (A)

65. Assertion (A) : Administration is now so vast an area that a philosophy of administration come close to being a philosophy of life.
Reason (R) : Administration is concerned with both ends and means.
Ans : (B)

66. Assertion (A) : Generalists play an important role in the policy function of the Government in India.
Reason (R) : A large number of top post in the Union and State Administrations are manned by the specialists.
Ans : (C)

67. Assertion (A) : The system approach is based on the concept that organisation should be studied as a whole because the behaviour of one part has a considerable impact on the others.
Reason (R) : The system approach ignores the infractions between a system and its environment.
Ans : (C)

68. Assertion (A) : The concept of civil service neutrality is changing in developing countries.
Reason (R) : Civil servants play a pivotal role in the developing countries.
Ans : (B)

69. Assertion (A) : Paul Appleby suggested two basic types of changes in administration constant and episodic.
Reason (R) : Constant change is reorganisation and episodic is incremental change.
Ans : (C)

70. Assertion (A) : The grapevine supplements the formal sources of information in the organisation. It has its usefulness in organisation life.
Reason (R) : It creates an actual system of relationship in the organisation which is vastly different from what is formally prescribe.
Ans : (B)

71. Assertion (A) : Administrative State is the name given to that State in which the executive branch is dominate one even though it has the legislature and the judiciary.
Reason (R) : The function Public Administration have expanded in scale, range and nature.
Ans : (B)

72. Assertion (A) : Open model theorists dislike the rigidity, the inflexibility, emphasis on means and anti-humanist view of bureaucracy.
Reason (R) : Close model theorists consider organisation as a fluid network structure.
Ans : (C)

73. ‘Bureaucracy is the term usually applied to a system of Government to control of which is so completely in the hands of officials that their power jeopardize the liberty of ordinary citizens.’
Who has given this statement ?
(A) H. J. Laski
(B) Ramsay Muir
(C) Lord Hewart
(D) Betrand Russel
Ans : (A)

74. Which one is major role of formal organisation ?
(A) Satisfaction of member
(B) Profitability of enterprises
(C) Service to the society
(D) Harmonious relations in the organization
Ans : (B)

75. Match list-I with list-II and select the correct answer—
List-I (Name of Writer)
(a) Paul H. Appleby
(b) Arther F. Bentley
(c) David Easton
(d) Wright Mills
List-II (Book)
1. Big Democracy
2. The Power Elite
3. The Process of Government
4. The Political system
Codes :
(a) (b) (c) (d)
(A) 1 4 2 3
(B) 1 3 2 4
(C) 1 3 4 2
(D) 1 2 4 3
Ans : (C)

76. Match list-I with list-II and select the correct answer—
List-I (Name of Thinker)
(a) Maslow
(b) Alderfer
(c) Pareek
(d) Porter and Lawler
List-II (Theory)
1. Need Hierarchy Theory
2. Mixed Model
3. Performance Satisfaction Theory
4. ERG Theory
5. Integrated Theory of Work Motivation.
Codes :
(a) (b) (c) (d)
(A) 1 4 2 5
(B) 1 4 5 3
(C) 1 3 5 2
(D) 1 4 5 2
Ans : (B)

77. A significant approach to communication event is based on Laswell formula, that deal with—
(A) Communication sequence
(B) Communication – interpretation modes
(C) Communication competence
(D) Communication control
Ans : (D)

78. Which one of following leadership styles permits substantial delegation of authority to subordinates and allows them to set objective and solve problems themselves ?
(A) Autocratic Leadership
(B) Democratic Leadership
(C) Permissive Leadership
(D) Visionary Leadership
Ans : (B)

79. The basis of influence in informal organisation is—
(A) Organisational position
(B) Person
(C) Authority
(D) Closeness with higher people
Ans : (B)

80. Which of the following book is called in apt text of Entrepreneurial Government ?
(A) Reinventing Government (David Osborne and Ted Gaeber)
(B) Government and Politics in Africa (Goren Hyden)
(C) A Public Management for all Seasons. (Christopher Hood)
(D) The Public Sector : Concept, Model and Approaches (Jan- Erik-Lan)
Ans : (A)

81. Which one of the following is not created by Act of Parliament ?
(A) University Grants Commission
(B) Backward Class Commission
(C) Atomic Energy Commission
(D) Railway Board
Ans : (D)

82. Match the list-I with list-II and select the correct answer—
List-I (Book)
(a) Organisations
(b) Hand book of Organisations
(c) Organisations in Action
(d) Principles of Organization
List-II (Author)
1. March
2. Mooney and Reiley
3. James D Thompson
4. March and Simon
Codes :
(a) (b) (c) (d)
(A) 2 3 4 1
(B) 1 2 3 4
(C) 2 4 3 1
(D) 4 1 2 3
Ans : (D)

83. Which one of the following administrative theories is called Genric Management ?
(A) Bureaucratic
(B) Scientific Management
(C) Human Relations
(D) Administrative Management
Ans : (C)

84. Which of the following is defined as a process of analysing and collecting information relating to operation and responsibilities of a specific job ?
(A) Job description
(B) Job design
(C) Job analysis
(D) Job enlargement
Ans : (C)

85. Match list-I with list-II and select the correct answer—
List-I
(a) Veblen
(b) Dewey
(c) Warnotte
(d) Lord Hewart
List-II
1. Professional Deformation
2. Occupational Psychosis
3. The great game of politics
4. Trained incapacity
5. New Despotism
Codes :
(a) (b) (c) (d)
(A) 4 3 1 5
(B) 4 2 1 5
(C) 2 1 5 2
(D) 3 2 4 5
Ans : (B)

86. To whom the Union Public Service Commission presents its annual report as to work done by the Commission ?
(A) Prime Minister
(B) Council of Minister
(C) Parliament
(D) President
Ans : (D)

87. On which date the Governor of Rajasthan assented The Rajasthan Panchayati Raj Act ?
(A) April 23, 1994
(B) May 21, 1994
(C) July 12, 1994
(D) October 2, 1994
Ans : (A)

88. Consider the following advantages claimed by the Centre for All India Services—
1. Control over State Administration.
2. Uniformed and highly standard of recruitment and training.
3. Experience is beneficial to both Centre and State.
4. Develop all India out-look.
Which are correct in above ?
(A) 1, 2 and 3
(B) 2, 3 and 4
(C) 1, 3 and 4
(D) 1, 2 and 4
Ans : (B)

89. When was the first Central Service Commission set up ?
(A) 1919
(B) 1926
(C) 1935
(D) 1937
Ans : (B)

90. Who had coined the term All India Service ?
(A) Aitchon Commission, 1887
(B) Committee on Division of Function, 1918
(C) Government of India, 1919
(D) Maxwell Committee, 1937
Ans : (B)

91. Who was called the Father of Local Self Government during British Rule ?
(A) Mayo
(B) Montagu
(C) Ripon
(D) Chelmsford
Ans : (C)

92. Which one is not mission of organisation development ?
(A) Improve the ability of individual member
(B) Avoid human emotions
(C) Reduce the tension
(D) Team management
Ans : (B)

93. For which of the following the President’s recommendation is required for introduction of Bill ?
(A) Bill for formation of new states and alternation of areas or name of existing State
(B) Money Bills
(C) Both (A) and (B)
(D) None of above
Ans : (C)

94. Which one of the following country first developed scientific method for recruitment ?
(A) Ancient India
(B) U.S.A.
(C) Britain
(D) China
Ans : (D)

95. ‘Administration is a moral act and administrator is a moral agent.’ Who said it ?
(A) Adm Brook
(B) Dimock
(C) C.A. Bernard
(D) Ordway Tead
Ans : (D)

96. Rajasthan Panchayati Raj Act has made detailed provisions for Gram Sabha. Which Articles of above Act is related with Gram Sabha ?
(A) Article 1, 2, 3, 4, 5
(B) Article 2, 3, 4, 5, 6
(C) Article 3, 4, 5, 6, 7, 8
(D) Article 4, 5, 6, 7, 8
Ans : (C)

97. There will be one or more Vigilance Committees to supervise the functions, implementation of schemes and other activities of Gram Panchayat in Rajasthan. Who will constitute the Vigilance Committee ?
(A) State Government
(B) District Collector
(C) Panchayat Samiti
(D) Gram Sabha
Ans : (D)

98. The middle-tier of Panchayati Raj Institution in Rajasthan is Panchayat Samiti, which is composed by—
1. Direct elected members for Panchayat Samiti.
2. Sarpanchs of Village Panchayats.
3. Member of State Legislative Assembly whose constituency partly or fully come
under the area of Panchayat Samiti.
4. Nominated members.
Codes :
(A) 1 and 3
(B) 2 and 4
(C) 2, 3 and 4
(D) 1, 2, and 4
Ans : (A)

99. Consider the following with reference to Rajasthan Panchayati Raj Act—
1. Direct Elections of members at all levels.
2. Reservation of seats for weaker-sections.
3. Direct Elections of all chairperson at all levels.
4. Elections are managed by State Election Commission.
Which of above are existed in Rajasthan ?
Codes :
(A) 1, 3, 4
(B) 1, 2, 4
(C) 1, 2, 3
(D) 2, 3, 4
Ans : (B)

100. The tenure system for Civil Services in India was started during the period of—
(A) Bentinck
(B) Cornwallis
(C) Curzon
(D) Montagu
Ans : (C)

SSC COMBINED GRADUATE LEVEL PRE EXAM 2008

1. Why two thin shirts can keep us warmer than a single thick shirt in winter?
(A) Two thin shirts become thicker so prevent transmission of heat
(B) Air layer between two shirts works as good conductor
(C) Air layer between two shirts behaves like insulating media
(D) No radiation of heat takes place
ANS (C)

2. Which layer of the earth's atmosphere reflect back the radio waves to the earth's surface ?
(A) Ionosphere
(B) Stratosphere
(C) Mesosphere
(D) Exosphere
ANS (A)

3. Sound can not pass through-
(A) Water
(B) steel
(C) Air
(D) Vacuum
ANS (D)

4. Water is not effective in extinguishing a fire caused by petrol because
(A) the flame is too hot for water to cool it down
(B) water and petrol react chemically
(C) water and petrol are miscible with each other
(D) water and petrol are immiscible with each other and petrol which forms the upper layer continues to burn
ANS (D)

5. The gas used in artificial ripening of fruits is
(A) Acetylene
(B) Methane
(C) Ethane
(D) Butane
ANS (A)

6. Ruby and sapphire are oxides of
(A) Copper
(B) Tin
(C) Iron
(D) Aluminium
ANS (D)

7. In the 53rd National Film Awards, the award for "Best Feature Film" has gone for
(A) Kaalpurush-Memories in the mist
(B) Rang De Basanti
(C) Parzania
(D) Paheli
ANS (A)

8. Which of the following pairs is incorrect ?
(A) Amirkhusro -- Sarod
(B) Bhim Sen Joshi -- Vocal music
(C) Utpal Dutt -- Films
(D) Shambhu Maharaj -- Kathak
ANS (A)

9. In a photocell light energy is converted into
(A) Potential energy
(B) Chemical energy
(C) Heat Energy
(D) Electrical enrgy
ANS (D)

10. The world's largest biofuel plant with production capacity of 110 million litres of fuel annually has been set up in
(A) China
(B) India
(C) Brazil
(D) U.S.A.

11. Which of the following Universities is a Central University ?
(A) Kolkata
(B) Dibrugarh
(C) Pondicherry
(D) Kurukshetra
ANS (C)

12. According to WHO, the bird flu virus cannot be transmitted through food cooked beyond
(A) 60 degree celsius
(B) 70 degree celsius
(C) 90 degree celsius
(D) 100 degree celsius
ANS (D)

13. The science of map-making is
(A) Cartography
(B) Geography
(C) Carpology
(D) Geology
ANS (A)

14. Who is known as the father of India missile technology ?
(A) Dr. U.R.Rao
(B) Dr. A.P.J.Abdul Kalam
(C) Dr. Chidambaram
(D) Dr. Homi Bhabha
ANS (B)

15. Which of the following cities / towns lies to the northern most latitude ?
(A) Patna
(B) Allahabad
(C) Panchmarhi
(D) Ahmedabad
ANS (A)

16. Who is the author of "A River Sutra" ?
(A) V.S.Naipaul
(B) Nirad C.Choudhuri
(C) Gita Mehta
(D) Vikram Seth
ANS (C)

17. Where was Asia Pacific Economic Cooperation (APEC) Summit, 2007 held ?
(A) New Delhi
(B) New York
(C) Singapore
(D) Sydney
ANS (D)

18. The capital of IMF made up by contribution of the
(A) Credit
(B) Deficit financing
(C) Member nations
(D) Borrowings
ANS (C)

19. Where is the National Academy of Agricultural Research Mangament located ?
(A) Dehradun
(B) Hyderabad
(C) New Delhi
(D) Itanagar
ANS (B)

20. Which kind of power accounts for the largest share of power generation in India ?
(A) Hydro-elctricity
(B) Thermal
(C) Nuclear
(D) Solar
ANS (B)

21. Age of a candidate to contest parliamentary election should not be lesser than
(A) 18 years
(B) 21 years
(C) 25 years
(D) 26 years
ANS (C)

22. Who develops the idea that "means justify the ends"?
(A) Kautilya
(B) Raja Ram Mohan Roy
(C) Swami Dayanand Saraswati
(D) Mahatma Gandhi
ANS (D)

23. wHO said that 'the real seat of taste is not the tongue , but the mind'?
(A) Aurobindo Ghosh
(B) Mahatma Gandhi
(C) Bal Gangadhar Tilak
(D) Swami Vivekananda
ANS (B)

24. Provincial autonomy aws one of the important features of the Act of
(A) 1935
(B) 1919
(C) 1909
(D) 1885
ANS (A)

25. Name the clan Buddha belongs to
(A) Gnathrika
(B) Maurya
(C) Saka
(D) Kuru
ANS (C)

Railway Recruitment Board Solved Paper 2008

Q.1. When a ray of light passes from an optically denser medium to a rarer medium, it
(a) Remains undeviated (b) Bends towards normal
(c) Bends away from normal (d) None of these

Q.2.Who is the author of "Anandmath" ?
(a) Rabindranath Tagore (b) Bankim chandra Chattopadhyaya
(c) Sarojini Nayadu (d) Jyotiba Phule

Q.3. Thimpu is the capital of -
(a) Nepal (b) Bhutan
(c) Thailand (d) Myanmar

Q.4. The time period of a seconds pendulum is
(a) 1 second (b) 4 seconds
(c) 3 seconds (d) 2 seconds

Q.5. The nuclear fuel in the sun is
(a) Helium (b) Uranium
(c) Hydrogen (d) Oxygen

Q.6. The second’s hand of a watch is 2 cm long. The velocity of its tip is
(a) 0.21 cm/sec. (b) 2.1 cm/sec.
(c) 21 cm/sec. (d) None of these

Q.7. In diesel engine, ignition is caused by
(a) Spark (b) Automatic starter
(c) Compression (d) Friction

Q.8. The mass-energy equivalence, relationship E = mc2 was propounded by
(a) Max Plank (b) Einstein
(c) Newton (d) Hertz

Q.9. The filament of bulb is made of
(a) Tungsten (b) Iron
(c) Nichrome (d) Carbon

Q.10. Anti-knocking can be lessened by
(a) Iso Octane (b) N Heptane
(c) TEL (d) Benzene

Q.11. In which of the following oxidation shows a positive oxidation state.
(a) CO (b) N2O
(c) NO (d) F2O

Q.12. Which of the following is used in photography?
(a) Silver Bromide (b) Sodium Bromide
(c) Potassium Chloride (d) Sodium Sulphate

Q.13. Which of the following is used in accumulator cell?
(a) Copper (b) Iron
(c) Lead (d) Zinc

Q.14. Choose the wrong statement :
(a) Single magnetic poles can exist
(b) Magnetic poles are always of equal strength
(c) Like poles repel each other
(d) None of these

Q.15. Laws of electrolysis are given by:
(a) Farady (b) Maxwell
(c) Lenz (d) Bohr

Q.16. Flemings left hand rule is used to fine out:
(a) Direction of magnetic field due to flow of current
(b) Direction of induced current due to effect of magnetic field
(c) Direction of motion of a current carrying conductor in magnetic field
(d) None of these

Q.17. The pH of a neutral solution at 250C is :
(a) 0 (b) 1.0
(c) 7.0 (d) 1.4

Q.18. The raw material used for the manufacture of Portland cement is :
(a) Limestone and clay (b) Alumina, clay and gypsum
(c) Gypsum and limestone (d) Gypsum and clay19. CaOCI2 is the chemical formula for a compound commonly known as :(a) Soda Ash (b) Lime
(c) Bleaching Powder (d) Plaster of Paris

Q.20. The glass used for making laboratory apparatus is :

(a) Pyrex glass (b) Hard glass
(c) Soft glass (d) Safety glass

Q.21. The iron produced in blast furnace is :
(a) Pig iron (b) Wrought iron
(c) Stainless steel (d) Steel

Q.22. Formation is a 40% solution of :
(a) Methanol (b) Methenal
(c) Methanoic acid (d) None of these

Q.23. Which of the following is not an ore of aluminium :
(a) Cryolite (b) Feldspar
(c) Bauxite (d) Azurite

Q.24. Rhombic monoclinic and plastic sulphur are:
(a) Isomers (b) Isotopes
(c) Allotropes (d) Hydrides of sulphur

Q.25. The alkaline hydrolysis of oils or fats gives soap and :
(a) Glycerol (b) Ethenol
(c) Glycol (d) Ethanoic acid

Q.26. The sight of a delicious food usually makes month watery. It is a :
(a) Hormonal response (b) Neural response
(c) Optic response (d) Olfactory response

Q.27. Nitrogen fixing bacteria are normally found in :
(a) Parasitic plants (b) Epiphytic plants
(c) Leguminous plants (d) Aquatic plants

Q.28. Powdery mildew of wheat is caused by:
(a) Bacteria (b) Virus
(c) Fungi (d) Protozoans

Q.29. Septic sore throat is caused by :
(a) Bacteria (b) Virus
(c) Fungi (d) Protozoans

Q.30. A person of blood group AB can give blood to :
(a) ‘A’ and ‘B’ (b) Only ‘AB’
(c) ‘A’, ‘B’ and ‘O’ (d) All of these

Q.31. Cell activities are controlled by :
(a) Chloroplast (b) Nitochondria
(c) Cytoplasm (d) Nucleus

Q.32. Which of the following helps eye to adjust the focal length of the eye lens :
(a) Cornea (b) Conjunctiva
(c) Ciliary body (d) Iris

Q.33. When pollen of a flower is transferred to the stigma of the same plant, pollination type is referred to as :
(a) Autogamy (b) Allogamy
(c) Xenogamy (d) Geitonogamy

Q.34. Respiration is a :
(a) Catabolic process (b) Anabolic process
(c) Both of these (d) None of these

Q.35. The structural and functional unit of kidneys are :
(a) Neurons (b) Nephrons
(c) Medula (d) None of these

Q.36. Which of the following enzymes is generally nor present in adult human :
(a) Renin (b) Pepsin
(c) Trypsin (d) Amylopsin

Q.37. The part of the plant which is responsible for carrying water and solutes from roots to various parts of plants is :
(a) Phloem (b) Xylem
(c) Duodenum (d) Sclercids

Q.38. ‘Widal test’ is used for susceptibility of :
(a) Malaria (b) Typhoid
(c) Cholera (d) Yellow fever

Q.39. Exchange of gases in plants takes place through :
(a) Stomata (b) Lenticels
(c) Cuticle (d) All of these

Q.40. ‘Lieutenant’ in army is equivalent to following rank in navy :
(a) Lieutenant (b) Sub Lieutanant
(c) Lieutenant Commander (d) None of these

Q.41. Pankaj Advani is associated with :
(a) Snooker (b) Golf
(c) Shooting (d) Archery

Q.42. Which city in the world is known as the forbidden city :
(a) Aberdeen (b) Jerusalem
(c) Mecca (d) Lhasa

Q.43. Rajeev Gandhi Khel Ratna Award carries a sum of :
(a) Rs. 3 lakns (b) Rs. 5 lakhs
(c) Rs. 1 lakh (d) Rs. 7 lakhs

Q.44. Who is known as ‘Nightingale of India’ :
(a) Asha Bhonsle (b) Begum Akhtar
(c) Sarojini Naidu (d) Vijaylaxmi Pandit

Q.45. Which one of the following is not included in the UNESCO’s list of world heritage site :
(a) Kaziranga National Park (b) Qutab Minar
(c) Champaner Pavagarh (d) None of these

Q.46. Which of the following is not an electric resistant?
(a) Lead (b) Ebonite
(c) Charcoal (d) Lac

Q.47. Which of the following is a complementary colour?
(a) Blue (b) Yellow
(c) Magenta (d) Yellow and Magenta

Q.48. The velocity of sound increases in air by ……… for every degree Celsius increase of temperature.
(a) 60 m/sec. (b) 0.61 m/sec.
(c) 60 ft/sec. (d) 0.61 km/sec.

Q.49. Henry is a unit of
(a) Capacity (b) Magnetic field
(c) Inductance (d) Frequency

Q.50. The velocity of rotation of Earth is
(a) 28 km/min (b) 30 km/min
(c) 25 km/min (d) 39.5 km/min

Q.51. What is Hubble?
(a) Warship (b) Star
(c) Telescope (d) Missile

Q.52. Which acid is normally found in lemon and grape fruits?
(a) Citric acid (b) Tartaric acid
(c) Ascorbic acid (d) Lactic acid

Q.53. At which temperature Fahrenheit and Celsius show same reading?
(a) -400 (b) 00
(c) -574.250 (d) 2730

Q.54. Limestone is metamorphosed to form
(a) Graphite (b) Quartz
(c) Granite (d) Marble

Q.55. Rift valley is formed by
(a) Earthquake (b) Folding
(c) Faulting (d) All of these

Q.56. If a piece of ice at 00C is mixed with water at 00C, then
(a) Whole ice melts (b) Some ice melts
(c) No ice melts (d) Temperature decreases

Q.57. Mettur Dam is built on the river
(a) Krishna (b) Cauvery
(c) Narmada (d) Mahanadi

Q.58. Mahendra Giri peak is situated in the
(a) Eastern Ghats (b) Western Ghats
(c) Shiwaliks (d) Vindhyachal

Q.59. In what time a sum will double itself at 20% per annum rate of interest?
(a) 2 years (b) 3 years
(c) 4 years (d) 5 years

Q.60. The famous Dilwara Temple is situated in
(a) Madhya Pradesh (b) Maharashtra
(c) Gujarat (d) Rajasthan

Q.61. Pneumonia affects
(a) Lungs (b) Tongue
(c) Liver (d) Kidney

Q.62. The chemical name of Vitamin A is
(a) Retinol (b) Jhiamin
(c) Biotic (d) Riboflavin

Q.63. A 130m long train crosses a bridge in 30 seconds at 45 kmph. The length of the bridge is
(a) 200m (b) 225m
(c) 245m (d) 250m

Q.64. By selling an article at some price a person gains 10%. If the article is sold at twice of the price, the gain percent will be
(a) 20% (b) 60%
(c) 100% (d) 120%

Q.65. Atoms are electrically charged as
(a) Positive (b) Negative
(c) Bi-positive (d) Neutral

Q.66. Ionic bond exists in
(a) Kcl (b) H2O
(c) NH3 (d) Cl3

Q.67. Which force is required to increase the momentum of an object to 40 kg m/s in 5 seconds?
(a) 2N (b) 4N
(c) 8N (d) 10N

Q.68. The river that does not form delta is
(a) Mahanadi (b) Tapti
(c) Krishna (d) Cauvery

Q.69. The ratio of the redius and height of a cone is 5 : 12, respectively. Its volume is 23147cc. Find its slant height.
(a) 13cm (b) 14cm
(c) 17cm (d) 26cm

Q.70. 27 students took part in a debate of a college. What is the probability that at least 3 of them have their birth days in the same month?
(a) 3/27 (b) 3/12
(c) 12 (d) 1

Q.71. Who was the first winner of Dada Saheb Phalke Award?
(a) B.N.Sarkar (b) Prithvi Raj Kapoor
(c) Devika Rani (d) Kanan Devi

Q.72. How many members are nominated by the President in the Rajya Sabha?
(a) 12 (b) 15
(c) 10 (d) 20

Q.73. Who was the founder of Brahmo Samaj?
(a) Raja Rammohan Roy (b) Aurobindo Ghosh
(c) Vivekanand (d) Dyanand Saraswati

Q.74. The Parliamentary System in India has been taken from
(a) America (b) Britain
(c) Australia (d) Ireland

Q.75. The electric supply in India was first started in
(a) Kolkata (b) Chennai
(c) Mumbai (d) Darjeeling

Q.76. Bhatnagar Prize is given in the field of
(a) Peace (b) Music and dance
(c) Science & technology (d) Fine arts

Q.77. Who discovered the solar system?
(a) Copernicus (b) Kepler
(c) Aryabhatta (d) Newton

Q.78. The Nobel Prize in Economics was started in
(a) 1901 (b) 1936
(c) 1957 (d) 1967

Q.79. In India, the second largest in
(a) Bengali (b) Urdu
(c) Telugu (d) Marathi

Q.80. The first Asian Games were held in
(a) Manila (b) Tokyo
(c) Jakarta (d) New Delhi

Q.81. The Islam was established in
(a) 7th A.D. (b) 5th A.D.
(c) 3rd B.C. (d) 5th B.C.

Q.82. The Olympic Games 2012 will be held in
(a) Moscow (b) Paris
(c) New York (d) London

Q.83. The main factor of air pollution is
(a) Lead (b) Copper
(c) Zinc (d) Gold

Q.84. The bauxite ore is found in
(a) Musabani (b) Karnapura
(c) Koderma (d) Palamu

Q.85. The first fertilizer plant in India was established in
(a) Trombay (b) Nangal
(c) Alwaye (d) Sindri

Q.86. Which of the following had strong navy?
(a) Bahmani (b) Chalukya
(c) Chola (d) Rashtrakuta

Ans-(C)

Q.87. The last king of Maurya Dynastry was
(a) Brihdarth (b) Kunal
(c) Samprati (d) Salishuk

Ans-(A)

Q.88. How will the image formed by a convex lens be affected if the central part of the lens is covered by black paper?
(a) Remaining part of the lens will form no image
(b) The central position of the image is absent
(c) There will be no effect
(d) The full image will be formed with lessened brightness

Ans-(D)

Q.89. The force between two parallel wires conducting current is used to define
(a) Ampere (b) Coulomb
(c) Volt (d) Newton
ANS - (a)

Q.90. A body is thrown vertically upward and it reaches 10m high. Find the velocity with which the body was thrown? (g = 9.8 m/s)
(a) 10 m/sec. (b) 18 m/sec.
(c) 14 m/sec. (d) 7 m/sec.
ANS - (c)

1. (c) 2. (b) 3. (b) 4. (d) 5. (c) 6. (a) 7. (c) 8. (b) 9. (a) 10. (a) 11. (d) 12. (a)
13. (c) 14. (a) 15. (a) 16. (c) 17. (c) 18. (b) 19. (c) 20. (a) 21. (a) 22. (b) 23. (d)
24. (c) 25. (a) 26. (d) 27. (c) 28. (c) 29. (a & b) 30. (b) 31. (d) 32. (c) 33. (a) 34. (a)
35. (b) 36. (a) 37. (b) 38. (b) 39. (d) 40. (b) 41. (a) 42. (d) 43. (b) 44. (c) 45. (d) 46. (a)
47. (d) 48. (b) 49. (c) 50. (a) 51. (c) 52. (a) 53. (a) 54. (d) 55. (c) 56. (c) 57. (b) 58. (a)
59. (d) 60. (d) 61. (a) 62. (a) 63. (c) 64. (d) 65. (d) 66. (a) 67. (c) 68. (b) 69. (a) 70. (c)
71. (c) 72. (a) 73. (a) 74. (b) 75. (d) 76. (c) 77. (a) 78. (d) 79. (c) 80. (d) 81. (a) 82. (d)
83. (a) 84. (d) 85. (d) .

L.I.C. Assistant Recruitment Test Previous year

English Language :Solved Model Paper
Directions—(Q. 1 to 10) Read the following passage carefully and answer the questions given below it. Certain words/phrases are printed in bold to help you to locate them while answering some of the questions.
Off the coast near Jeddah about six years ago I slipped a pair of flippers onto my feet, put on a face mask and gingerly poked my head beneath the placid surface of the Red Sea. I am not sure, now, exactly what I saw in that first glimpse; shafts of sunlight, probably, slanting off through clear blue water; a coral reef of fantastic beauty and, no doubt, many fish. But I do recall that by the time I surfaced I had already developed a need to return to that incredibly lovely world below. So I did. Indeed, it would not be much of an exaggeration to say that except for such periods as were necessary to earn a living, I have rarely been anywhere else. Diving became a passion and then a way of life—one, as you will gather, I totally endorse.

For the first year after that initial experience I amused myself by hunting fish with a spear gun. It was not at all difficult. Fish are so abundant there that divers do not need to use tanks to get down to where the fish are; they find them near the surface. Furthermore, the water of the Red Sea, like the Caribbean, the South Seas and parts of the Indian Ocean, is so transparent that you can see upto 150 feet away. Thus, all you really need are a mask, a snorkel tube to breathe through, flippers and a spear gun.

Hunting, however, began to pall on me. I began to wonder if it wouldn’t be more interesting—and more sporting—to photograph some of these magnificent creatures rather than kill them. It was certainly an ideal place for underwater photography. In addition to hundreds of species of fish the Red Sea coast offers thousands of miles of what they call ‘fringing reefs’—great barriers of coral 10 to 200 miles wide that wind along the African shore from Egypt to Djibouti and down the Asian shore from Aqaba to the Babal-Mandab at the gate of the Indian Ocean. Although no more than the accumulation of billions of coral polyps—minute creatures that produce a calcareous deposit—the reefs have grown to fantastic sizes, the largest being the 1,200 mile-long Great Barrier Reef in Australia.

1. In what way is the Red Sea SIMILAR to the Caribbean Sea, the South Seas, etc.
(1) Variety of Fish
(2) Fantastic Reefs
(3) Clear and transparent water
(A) 1 only
(B) 2 only
(C) 3 only
(D) All the three
(E) None of these

2. Which of the following changes occurred in the author after the initial period of a year or so ?
(1) He got fed up with hunting
(2) He struck with the idea of shooting the fish with a camera rather than by spear gun
(3) Hunting became more interesting and sporting
(A) 1 only
(B) 2 only
(C) 1 and 3 only
(D) 1 and 2 only
(E) All the three

3. Fishing in the Red Sea off the coast near Jeddah was—
(A) Difficult because the water was transparent
(B) Risky because tanks are needed to spot the fish
(C) Easy because the fish are clearly visible near the surface itself
(D) Cumbersome because of the tools like mask, snorkel tube, flippers, etc.
(E) More difficult than at the Caribbean Sea, the South Sea and the Indian Ocean

4. What are ‘fringing reefs’ ?
(A) Calcareous deposit formation
(B) Collection of fish
(C) Sea-food accumulation
(D) Collection of minute creatures other than fish
(E) None of these

Directions—(Q. 5 to 7) Which of the following is most nearly the SAME in meaning as the word given in bold as used in the passage ?

5. GINGERLY
(A) Cautiously
(B) Recklessly
(C) Clearly
(D) Purposely
(E) Unintentionally

6. MAGNIFICENT
(A) Delicate
(B) Fragile
(C) Obvious
(D) Powerful
(E) Splendid

7. TRANSPARENT
(A) Opaque
(B) Glowing
(C) Dependable
(D) Visible
(E) Dark

Directions—(Q. 8 to 10) Which of the following is most OPPOSITE in meaning of the word given in bold as used in the passage.

8. RECALL
(A) Call up
(B) Recollect
(C) Forget
(D) Invite
(E) Send back

9. MINUTE
(A) Enormous
(B) Tiny
(C) Small
(D) Heavy
(E) Delay

10. FANTASTIC
(A) Incredible
(B) Plausible
(C) Unforgettable
(D) Imaginary
(E) Realistic

Directions—(Q. 11 to 15) In each question a sentence is given with a part printed in bold. That part may contain a grammatical error. Replace that part with the four choices given (A), (B), (C) and (D). If the sentence is correct and no correction is required, mark (E) as the answer.

11. In quick time she got acquainted with the new environment.
(A) In enough time
(B) In small time
(C) On time only
(D) In no time
(E) No correction required

12. The meeting was postponed due to lack of quorum.
(A) Due
(B) Because of
(C) For
(D) Against the
(E) No correction required

13. It was quite clear that the athlete can be able to improve upon his own record.
(A) Will be able to
(B) Should be able
(C) Would be able
(D) Be able
(E) No correction required

14. He has not written any book since his mother had died.
(A) Died
(B) Have died
(C) Has died
(D) Was dead
(E) No correction required

15. It was too cold to go out last evening, so we all stayed at home.
(A) Too cold for going
(B) Very cold to go
(C) Extremely cold for go
(D) So cold that to go
(E) No correction required

Directions—(Q. 16 to 20) Read each sentence to find out whether there is any grammatical error or idiomatic error in it. The error, if any, will be in one part of the sentence. The letter of that part is the answer. If there is no error, the answer is (E).
(Ignore errors of punctuation, if any).

16. Since the online education system (A) / has been poorly designed there (B) / are not many
institute (C) / that offer this facility.(D) No error (E)

17. Unless the new resolution is (A) / completely reviewed it (B) / will cause a great deal of (C) / trouble for new employees. (D) No error (E)

18. After careful scrutiny of the report (A) / variety mistakes that had been (B) / made by the research (C) / department were found. (D) No error (E)

19. His industrious nature (A) /and calm temperament (B) / have endeared him (C)/ to his colleagues and one’s superiors. (D) No error (E)

20. The celebrities that organized (A)/the marathon were aiming (B)/ to create an awareness (C)/ about the treatment of diabetes. (D) No error (E)

Directions—(Q. 21 to 23) Choose the word/phrase which is most nearly the SAME in meaning as the word printed in bold as used in the passage.

21. Incurred
(A) collected
(B) included
(C) spent
(D) experienced
(E) adjusted

22. Prevailing
(A) popular
(B) implemented
(C) existing
(D) persuading
(E) winning

23. Hurdles
(A) bars
(B) defects
(C) rejections
(D) protests
(E) obstacles

Directions—(Q. 24 and 25) Choose the word/phrase which is most OPPOSITE in meaning of the word printed in bold as used in the passage.

24. Vital
(A) practical
(B) voluntary
(C) negative
(D) worthless
(E) minimum

25. Flourishing
(A) drooping
(B) declining
(C) fluctuating
(D) opposing
(E) lacking

Directions—(Q. 26 to 30) In each sentence below, one word has been printed in bold. Below the sentence five words are suggested, one of which can replace the word printed in bold without changing the meaning of the sentence. Find out the appropriate word in each case.

26. The professor requested the Dean to visit his house.
(A) get in
(B) welcome
(C) see
(D) call on
(E) call at

27. She knew that he will not be able to slip past the watchful custom officials.
(A) fall
(B) run
(C) hide
(D) chit
(E) escape

28. At night Romesh complained of severe pain in his stomach.
(A) serious
(B) deep
(C) bad
(D) biting
(E) intense

29. Shivaji always excelled in the use of the sword.
(A) surpassed
(B) speeded
(C) ahead
(D) skilled
(E) defeated

30. You must stay with your son when he is ill.
(A) reside
(B) remain
(C) stop
(D) occupy
(E) rest

Directions—(Q. 31 to 35) In each question below, a sentence is given with a part of it printed in bold type. That part may contain a grammatical error. Each sentence is followed by phrases (A), (B), (C) and (D). Find out which phrase should replace the phrase given in bold to correct the error, if there is any, and to make the sentence grammatically meaningful and correct. If the sentence is correct as it is and no correction is required mark (E) as the answer.

31. Fishing and swimming are two different activities, independence of one another.
(A) independent of the other
(B) independence of the other
(C) independent of each other
(D) interdependence on each other
(E) No correction required

32. An early action on our suggestion, preferably before the elections are announced, will be appreciative.
(A) would be appreciate
(B) would have been appreciate
(C) would have been appreciated
(D) will be appreciated
(E) No correction required

33. He is the man whose advice is difficult in following.
(A) advice is not easy in following
(B) advice is difficult to follow
(C) advice has difficult to follow
(D) advice has difficulty to follow
(E) No correction required

34. He told me that he only had a little money.
(A) tells me that he only has a little
(B) told me that only he has a little
(C) only told me that he has little
(D) told me that he had only a little
(E) No correction required

35. You must ensure that I get my cheque encash before Saturday.
(A) my cheque cashed
(B) cash my cheque
(C) my cheque cash
(D) encash my cheque
(E) No correction required

Directions—(Q. 36 to 40) Pick out the most effective word from the given words to fill in the blank to make the sentence meaningfully complete.

36. He tends to ………… to any suggestion I make in meetings.
(A) differ
(B) agree
(C) accept
(D) act
(E) cooperate

37. We were shocked by the young man’s…………for money.
(A) greed
(B) acumen
(C) versatility
(D) projection
(E) indifference

38. Let us cultivate a strong will, a …………mental desire and determination to achieve our ideals.
(A) tall
(B) sure
(C) sardonic
(D) keen
(E) cutting

39. He ………… children to open their eyes and ears to the beauty of life.
(A) admonished
(B) promised
(C) exhorted
(D) complemented
(E) reprimanded

40. We must work hard towards ………… of the underprivileged people of our country.
(A) proliferation
(B) emancipation
(C) contribution
(D) association
(E) unification

Answers
1. (C) 2. (A) 3. (C) 4. (A) 5. (A) 6. (E) 7. (D) 8. (C) 9. (A) 10. (E)
11. (D) 12. (C) 13. (C) 14. (A) 15. (E) 16. (C) 17. (D) 18. (B) 19. (D) 20. (C)
21. (C) 22. (C) 23. (E) 24. (D) 25. (B) 26. (C) 27. (B) 28. (E) 29. (A) 30. (A)
31. (C) 32. (D) 33. (B) 34. (D) 35. (D) 36. (B) 37. (A) 38. (D) 39. (C) 40. (B)

CIVIL SERVICES (Main) EXAM, 2007 Solved Paper

General Studies Paper--1


Q.1. Answer any one of the following Questions in about 250 words:

(A) What was the character of socio-religious reforms in 19th century and how did they contribute to national awakening in india ?
(B) The crisis of the colonial order during 1919 and 1939 was directly linked to constitutional reforms ,disillusionment and militant anticolonial struggles.Elucidate.
Ans. (A) The socio-religious reforms in 19th century were oriented towards restructuring the Indian Society along the modern lines and for growth of national unity and solidarity. Rationlism was the most significant element of these movements wherein socio-reliogious practices were judged onthe stand point of their social and contemporary relevence . The rationale behind taking together both social and religious aspects was the religious one being closely interwoven with social structure.The social norms were determined by religious tenets, hence both aspects had to be reformed.
Another important character was that they do not stand for structural transformation of society ,rather for changes in broad social framework. There was no total rejection of tradion nor was it blind imitation of westernization, it was to be on intellectual and creative process through which Indian culture and thought were to be renovated as postulated by Raja Rammohan Roy.Therefore its Key-notes were modernizingand rational approach. Reform movemwntalso sought to influence the political authority,administration and legislation as was evident in socio-cultural policy of colonial rulars. On the whole,these movements were progressive in nature as they aimed at attainment of scientifically engineered change and social salvation. The reform movements fostered a new consciousness--Social cultural and above all critical which led to introspection of traditional ethos and identificationof socio-religious evils.It revived the faith in indian glory, self respect andself pride which was supported by theosophical society.As a result of reformedoutlook,many indians began to acquire modern, secular and national outlook.It aimed at achieving the following objectives--emancipation of women and extensionof equal rights to them,removal of cast rigidities esp.Untouchability and also to spread education. Numerous Individuals like Raja Rammohan Roy,Pt.IshwerchandVidhyasagar,Henry V.Derozio,reform societies and religious organisations worked hard to spreededucation among women,to encourage widow remarriage,prevent childmarriage, to bring women out of purdah.The manifestation of the above could be seen in the active role played by women in freedom struggle.
As modern democratic rationalist ideas spread among indians they vehemently opposed caste system.Brahmo Samaj,Arya Samaj,Prarthna Samaj,Ramkrishna Mission and nearly all great reformers attecked it. As a result ofawakening,the lower castes rose in defence of their rights. Thus socio-religiousreform movements infused self pride and nationalistic outlook which greatly contributed to national awakening.

(B)By 1919,the colonial rule was not able to fulfil the demands and aspirationsof indians,who felt cheated by promises made by the british during first world warand were disillusioned with the exploitative designs of rowllat Act and Jallianwala Bagh massacare.Urgent need was felt for constitutional reformes and the council act of 1919was introduced which advocated dual governance and expensionof legislature which further failed to satsfy the indian demands .Unrealised promisesled to rise of mass movements like non co-operation movement and later civil disobedience movement. These mass movements under the charismatic leadership of gandhiji, gave an expresssion of more aggressive and redical kind of nationalism.
The sudden withdrawal of non co-operationMovement and suspension of civil disobedience movement,shattered the faith of young congressmenand they lost allegiance to gandhian techniques of mass struggle.This gave rise to Revolutionary terrorism which had firm connection to overthrow british rule by striking terror and violence.Organisationslike hindustan republican Association and Hindustan Republican socialist Association organised action against British rule.
The Act of 1935 was also disappointing,as it failed to fulfil the purna swaraj demand of the nation.Pt. nehru described it was unwanted,undemocratic and anti national Act. Congress fought election (according to provisions of 1935 Act)and formed state Ministriesbut resigned in1937 on the issue of involvement in second world war which again created problems for the British Goverment. Therefore the period1919-1939 was a period of turmoil for colonial empire as its weakening administrative mechanism was evident in front of the rising tide of indian Nationalism.

Q.2 Answer any two of the following questions in about 150 words each:

(a) What are the salient features of Govt. of India Acts of 1858 and 1909?

(b) Do you think Mahatma Gandhi’s support to Khilaftat Movement had diluted his secular credentials? Give your argument based on the assessment of events.

(c) Evaluate the contribution of revolutionary terrorism represented by Bhagat Singh to the cause of India’s struggle for Independence?

Ans.(a) The following are salient features of –Act of 1858

1. The Act transferred the Indian Administration to British Crwon and changed the designation of Governor General to ‘The Vicorey of India’

2. The powers related to government and revenues of India, were to be exercised by secretary of state for India who would by aided by a council of 15 Members (8-nominated by Crown and 7 elected by Directors). The secretary of state was to be a member of British Cabinet and responsible to British Parliament.

3. The Council of India was to be an advisory body and was presided by secretary of state who was empowered to override the majority. However, he could not do so in financial matteres.

4. The Act empowered the crown to appoint the Viceroy of India and Governors of presidencies. The Viceroy was empowered to appoint Lt. Governors with crown’s approval.Act of 1909

1. Expansion of Councils : The number of additional members (legislative) in Viceroy’s Executive Council was raised from 16 to 60, Legislative Councils of Madras, Bengal and Bombay were increased to 50 members each.

2. The legislative council consisted of four categories of members Ex officio members nominated officials, nominated no-officials and elected members. Official majority was retained in Imperial legislative council but it was given up in provinces.

3. Introduction of Communal Electorates- Muslims were given 2 privileges-direct representation and right of plural voting.

4. The Act of 1909 for the fists time gave recognition to elective principle for appointment of non official members to councils but it was restricted and discriminatory franchise.

5. The members of the legislative council could discuss the budged and pass resolution and they could also ask supplementary questions.

6. The size of executive councils of Governors in Bombay and Madras increased to four and Indians were appointed in them.(b) There is no valid reason to believe that Mahatma Gandhi’s support to Khilafat Movement had diluted his secular credentials because Gandhi viewed the Khilafat agitation as golden opportunity for consolidation Hindu-Muslim unity and bringing the Muslim masses into the national movement. Gandhiji realised that India which is highly fragmented, different sects, religios ideology, race, and cultural people can only be uited through mass movement. This mass movement would provide them platform to fight for their own different demands and seeing that the alien regime stood in opposition to them. Gandhiji looked upon the Khilafat agitation as “an opportunity of uniting Hindus and Muslims as would not arise in a hundred years”. Muslim and Hindu bond got cemented. Muslim leaders gave a call to refrain from slaughtering cow on Bakra Id. Swami Shradhanand delivered a speech from Jama Masjid Delhi. Hindu and Muslim workers began to drill together. The English government declared it illegal. Gandhi involvement in the Khilafat movement had brought urban muslims into the nationalist movement and had been thus responsible in part for the feeling of nationalist enthusiasm and exhilatation that prevailed in the country in those days. Gandhiji to some extent failed in raising the religious political consciousness of the Muslims to the higher plane of secular political consciousness. At the same time it should also be kept in view that the Khilafat movement represented much wider feeling of the Muslims than their concern for the Caliph. It was in reality an aspect of the genereal spread of anti imperialist feelings among the Muslims. These feelings found concrete expression on the Khilafat questions which Gandhiji was well aware of.(c) Revolutionary terrorism emereged as an alternative political line- for moderated constitutional agitation, and extremists direct action represented by Bhagat Singh and his other comrades made a major advance in broadening the scope of repolution by postulation its first objective as national liberation and another to establish a new social order in which exploitation of man by man must end.Bhagat Singh defined Socialism in a scientific manner-abolition of capitalism and class domination. He advocated Religion as one’s private conceren and communalism as an enemy to be fought. In his article : Why I am atheist he subjected religious philosophy to a critique. Thus, the movement under Bhagat singh made the masses more progressive in outlook. He had already abandoned his belief in terrorism in 1929 and turned to socialism and believed that popular mass movement could alone lead to successful revolution. Therefore, he established Punjab Naujawan Bharat Sabha in 1926 to carry political work among youth, peasants and workers.Thsus it can be concluded that his deep patriotism, secularism and progressive ideas helped spread nationa and socialistic consciousness and made an abiding contribution to national freedom movement.

Q. 3 Write about the following (not exceeding 20 words each) :

(a) Age of Sangam Literature (b) Bhakti (c) Ashtadhyayi of Panini (d) Charvakas (e) Ajivikas (f) Gndhara Art (g) Malechchhas (h) Lingayats (i) Megasthenes (j) R.C. Dutt (k) Nagrjunakonda (l) Pastoralism (m) Rudramadevi (n) Sati (o) Ramanuja

Ans. (a) Earliest Tamil works composed during Pandya’s 500 BC-500 AD, throw light on political history, store house on social and religious history.

(b) Originated in South 7th- 9th AD by Tamil mystic saints, pioneer Ramanuja, in north Ramananda, believed in monotheism, local dialect, devotional worship, criticize caste system.

(c) Flourished in 5th BC, grammatical work, divided into eight sectors, contain 4000 sutras, dealing mainly with bhasya.

(d) Founder of Lokyata Darsana, propounds gross materialism, sense preceprion is the only source of knowldege, no conscious entity like soul or mind.

(e) Heterodox sect founded by Maskariputra Gosala sect saints known as Sudra Sanyasi. Atheist sect believes in universe conditioned and determined by destiny, prevalent during Bindusara.

(f) Originated 1-4th A.D. fusion of Greek, Roman, Chinese Persian and Indian style of art. Themeon Mahayana Buddhism, Patron Kushan and shakya rulers.

(g) For foreigners term used in Sanskrit; texts consider as barbarian like a Greeks, Sakas Kambhoj.

(h) Founded by Basava and his nephew Channa Basava, noteworthy for its social and cult doctrine, believe in qualified monism, against Brahmin dominace and rites.

(i) Ambassador of Seleucus Nicator in Chandra Gupta Maurya, wrote Indica, throws light on social economic and religious life during Mauryas.

(j) Earliest Civil Srvant wrote Economic history of India, President of INC in 1890, In this book he ahs elaborated his thesis of Marx.

(k) Capital of Iksvakas in Andhra Prades famous for Buddhist Stupa and Chaitya, belongs to Amarvati art school, named after great Buddhist scholaR Nagrjuna.

(l) It is important economic aspect of tribal organistation, breed and rear cattle for commercial pur- poses. Todas in Nilgiri and Gujjars in Himachal Pradesh and Jammu & Kashmir practice it.(m) Kakatiyas princess maintained intergrity of her empire, praised by Marco Polo, handed over power to Prataparudra II her grandson.(n) Social evil practised in ancient and medieval India especially among Kashtriya’s. First written evidence Eran inscription 510 AD. Banned in 1829 by William Bentinck.(o) His philosophy Vishista Advaitvada, Advocated Prapatti Marg, Saguna saint wrote Vedanta Sangraha popularly known as yatiraja, wrote commentary on Vedanta Sutra.

Q. 4. Answer any two of the following questions (in about 125 words each) :

(a) Explain how the Himalayan and the Tibetan highlands play an important role in the development of the South-West Monsoon ?

(b) Technological changes have brought in a major shift in the use of roads as transport corridors in India. How far do you agree with this view ?

(c) Explain the nature and causes of growing slums problems in the metropolitan cities of India.

Ans. (a) Himalayas from a natural boundary of the Indian Sub continent. Himalayas check the cold winds from Central Asia and prevents moisture laden winds from Central Asia and prevents moisture laden winds from crossing to North. As the Sun travels northwards there is change in wind circulation pattern both in lower and upper troposphere. At lower level due to the heating of land an Equational Low Pressure Zone is formed known as ITCZ (Inter Tropical Convergence Zone) which is formed approx parallel to the Himalayas. This zone determines the monsoon rain pattern in North India.Due to the heating of Himalayas and Tibetian Highland, Easterly jet stream orginates and generated tropical depression because Tibetian Highland and Himalayas are heated by Sun radiations. These radiations are reflected back to the atmosphere, which help in generation clockwise air circulation in the upper troposphere resulting in development of tropical depression. The intensity of South-West monsoon depends upon generation of these tropical depressions and formation ITCZs. The more tropical depression orginates, the more intense will be the South-West monsoon.(b) Transport system is a mirror of economic development and material prosperity of country. Several programmes have been implemented to develop roads as arteries of transport system like Nagpur Plan (1943), Twenty year Road Plane or Hyderabad Plan; Rural Development Plan; BOT, Centre Road Fund and Bharat Nirman. I fully agree with the above statement that technological changes have brought major shifts in the use of roads. This process has been evolutionary rather than revolutionary; new technology has been introduced in making all weather roads, width of roads increase for faster transportation, layering and concrete roads, better network of roads, increases in track capacity and weight, lower fuel consumptions. Proper management of corridors road, bridges and geotechniques involved in building tunnels and double road, use of innovative materials/Highway road safety norms have been made stringent and different methodology has been introduced like road facing, divider. Drainage of water; better elevation and right parameter of turns.(c) The rapid growth of slums in the metropolitan cities of India is due to various reasons like urban poverty, lack of remunerative employment opportunities, unequal regional development, lack of infrastructural development and rural employment, and lack of affordable legal shelter for the poor due to soaring land price in the urban areas.Development and opportunities for jobs attract a large number of migrants from rural areas. Increases in a greater proportion to the level of economic development slums swell, pressure on urban infrastructure intensifies and misery increases. It affects urban infrastructure like in Mumbai. Metti river was covered with slums; there was no drainage outlet left out resulting in flooding. Basic amenities, public utilities, transport and communication further deteriorate. It also enhances criminalization.Our planners’ lackadaisical attitude resulting in poor agricultural and rural industry in villages, lack of basic amenities like water, electricity medical and job opportunities are other causes. Even small town lack such opportunities to check migration.Economic development disparities will increase the exodus of rural people and mushrooming of more slums. Slums are also encouraged by politician to influence their clout.

Q. 5 Write notes on the following (in about 20 words each) 2x5 = 10

(a) Special Economic Zone (b) Inceptisol (c) Jarawas (d) Indira Point (e) Causes of Chambal Ravines

Ans. (a) Announced in 2000 by Ministry of Commerce to promote economic growth supported by quality infrastructure, fiscal package both at centre and state level.

(b) Soil usually moist, incomplete weathering, no clay, Iron and Aluminum oxide; saturated layers absent.
(c) Negrito tribes of Andaman; numerical thin strength, social contact futile due to their shy attitude, stay away from settling on Island.
(d) It is the farthest point of India located in Great Nicobar, also known as Pygmalion or Passon point; destroyed by Tsunami in 2006.
(e) Type of soil erosion gullies further deepen the grooves and result in ravines, barren land. It has resulted in poverty and unemployment and breeds crime like dacoity.

Q. 6 Answer any one of the following questions (in about 250 words).

(a) What is a Constitution? What are the main sources of the Indian Constitution?
(b) Difference between Directive Principles and Fundamental Rights. Discuss some of the measures taken by the Union and State Governments for the implementation of the Directive Principles of State policy.
Ans. (a) A Constitution is the supreme law of the land which reflects the fundamental principles on which the government of that country is based. It defines the organisation of the government, the distribution of powers to the various organs of the government, general principles on which those powers are exercised and the rights and duties of the citizens.
The main sources from which the Indian Constitution has been drawn are:
The most profound influence and material source is the Government of India Act 1935. The Federal Scheme, Judiciary, Governors’ Status, emergency powers, the Public Services Commission and most of the administrative details are drawn from this Act. More than half the provisions of our Constitution are drawn from it.
From the British our Constitution borrows the system of Parliamentary government, cabinet system, single citizenship, procedure and privileges of Parliament, prerogative writs.
The USA Constitution was the source for the provisions pertaining to Judicial review, Independence of Judiciary, Fundamental rights, removal of the Supreme Court and High Courts.
The Irish inspired the inclusion of the directive principles, method of election of the President and the nomination of members to Rajya Sabha. The Canadian Constitution is an example of Federation with a strong centre. The residuary powers of legislation are vested in the centre and the centre appoints the Governors from the Australia Constitution, concurrent list of legislative subjects and provisions regarding freedom of trade, commerce and intercourse, Weimer Constitution inspired to draw provision related to suspension of Fundamental rights during the proclamation of emergency.
Hence we can say our Constitution is conglomeration of the best features of the different Constitutions and modifies them according to our needs.

(b) Though the objectives of both FRs and DPs are identical there are still significant differences as follows.
FRs are justiciable, courts protect them and compel the state to respect them through Art. 32 and Art 226. But the Courts cannot direct the state to give effect to DPs as they are unjusticiable and cannot be enforced.
The FRs’ are mostly political rights which tend to restrain the state; they are legal in nature. They are also negative rights. Articles 14 and 21 are negatively worded. DPs are social rights in the form of positive obligations of the State and are morally binding upon the State.
There are numerous statutes both Central and State, which have their genesis in the desire of the government to implement DPs like Zamindar abolition Acts so that cultivator will become land owner. Agriculture Land Ceiling Acts were passed, several Amendments were made to implement DP principles i.e., the 73rd amendment to implement Panchayati Raj.
In 1969 nationalisation of banks, coalmines in 1973 inculcate Art 39 (b) and (c) legislation guaranteeing equal pay for equal work is relatable to Art 39 (d). The 26th Amendment of the Constitution made to abolish the Privy Purses which were granted to the rulers of Indian States. The 28th Amendment conferred on the Parliament the power to vary or revoke the conditions of service of the members of Indian Civil Service. Similarly ban on sale of liquor in Gujarat. All these were purported to implement the ideals enshrined in DPs.

Q. 7 Answer any one of the following (in 250 words) :

(a) What is regionalism? In which way has regionalism affected the Indian polity?
(b) What are the main determinants of voting behavior in India?

Ans. (a) Regionalism is the idea or practice of dividing a country into smaller units for political, economic, social and cultural purposes. Politically regionalism is linked to decentralization. It can also define as political ideology that focus on the interest of a particular region, or group of regions where people of particular faith or religion, or cultural affinity or linguistic affinity or caste or tribal affinity reside.
After independence because of various elements like regional development imbalance, non fulfillment of expectation of masses in terms of their aspirations with regard to development, river dispute or mineral rights, exploitation of people’s sentiments by political parties, exploitation of their culture or identity, linguistic hegemony are responsible for regionalism. Sometimes foreign powers also provoke regionalism to destabilize India’s growth and development.
Regionalism had manifold influence on Indian polity. In 1953 linguistic regionalism led to the formation of Andhra Pradesh Telgu speaking state. In southern States regional parties cropped up. Regionalism led to the recognition of regional languages as official languages of nation like Santhali, Konkani, It was regional feeling which was responsible for bifurcation of Punjab, Bombay, Assam. Sometimes regional imbalance in development initiates division like Uttar Pradesh into Uttarakhand, Madhya Pradesh into Chhattisgarh. Voices are raised to further divide UP into smaller States like Bundelkhand or Harit Pradesh.
Bodo Tribes in Assam and Gorkhas in West Bangal Demanded separate statehood due to regional inclination.
Regionalism has to some extent weakened the Centre Govt. and in framing its policies because it had to play appeasement policies. Country like India requires strong concrete policies to maintain integrity and unity of India.
Regional aspirations have been fulfilled over the period of time which no doubt has helped in maintaining unity amid diversity but in near future it can create new problems to india like rivers water dispute, distribution of electricity.

(b) Since Independence there are continuous changes in the voting behaviors in India. When India got independence, there was zeal for development and progressive growth. Congress was major party, opposition was highly fragmented. Then in South and East India Communist ideology emerged. Kerala and West Bengal Became strongholds of Communist voters. Another shift occurred due emergence of regional parties in the south India like DMK. Hence forth the voter became more inclined to regional ideology. From 1980 onward communalism, casteism, regional development slogan had drastically changed the voting pattern. In 1990 communalism and casteism further consolidated their roots and had deep impact upon voter behavior. Caste, religion, region, local cause and local charismatic leaders had deep impact upon voting behavior. National party influence decreased on voter behavior and national issues no longer influence voting behavior which it used to be 1950-70s. One thing which is noticeable is that the voter over the period of time has exerted his/her right and thrown out the governments for their incompetency.

Q. 8 Anser any two of the following questions (150 words) :

(a) Art. 74 States that president is bound to exercise his functions, in accordance with the advice of the council of ministers but there are few exceptions.
(b) What is Pro-tem Speaker?
(c) Under what circumstances may Parliament legislate on state subjects?
Ans. (a) The following are exceptions-
The appointment of the Prime Minister- While appointing /selecting the Prime Minister the President is to find out which party commands majority in the Lok Sabha. In case when there is no clear majority and various parties are combing to form coalition, then the President has to apply his mind in a judicious, fair and impartial manner.
· When the Prime Minster who has lost the confidence of the Lok Sabha, Council of Minsiters advises the President to dissolve the house, the President ahs to judge independently whether such a course of action would be in the national interest. The President should not bindly follow the advice tendered by the council of minister which does not enjoy the confidence of the lok sabha. If it is possible to form an alternative Government, the President should avoid dissolution.
· The president can ask council of minister to reconsider its advice this decision to be exercised independently by the President.
· The President can under Act 78 (b) call for information relating to administration and legislation.
· Under Act. 352 (3) the President has to ensure that during proclamation of Emergency decision of the cabinet must come in writing.

(b) After each general election the President nominates the senior most member elected as Speaker Protem. He performs the duties of office of the Speaker till the house elects one. Usually the only duty performed by the Speaker Pro-tem is to administer oath to the newly elected members. Pro-tem speaker is appointed both in Lok Sabha at the Union lelvel and Vidhan Sabha at the State level. In 2005 the Governor of Jharkhand appointed a junior member of the Assembly as Speaker. The Supreme Court expressed its disapproval.
(c) In normal times the distribution of powers must be strictly maintained and neither the state nor the centre can encroach upon the power allocated to be other by the Constitution yet there are certain exceptional circumstances under which the union Parliament can legislate on State Subjects.
1. According to Article 249 if the Rajya Sabha passes a resolution supported by 2/3 of the members present and voting that it is necessary or expedient in the national interest that Parliament should make laws with respect to any matter enumerated within state list, then it shall be lawful for the Parliament to make laws for the whole or any part of the territory of India with respect to that matter so long as the resolution remain in force. Such resolution lasts for one year, it may be renewed as many times as necessary but not exceeding a year at a time. These laws will however, cease to have effect on the expiration of the period of six months after resolution has ceased to operate.
2. During the Proclamation of Emergency, as per Art. 250 the Parliament shall have power to make laws for the whole or any part of the country with respect to all matters enumerated in the state list.
3. Parliament’s power to legislate with the consent of the States.
According to Art 52 if the state legislature of two or more states pass resolution to the effect that it is desirable to have a law passed by the Parliament on any matters in the state list, it shall be lawful for Parliament to make laws regulation that matter.
4. Parliament’s power to legislate for giving effect to treaties and international agreements.
Act. 253 empowers the Parliament to make any law for the whole or any part of the territory of India for implementing treaties and international agreements and conventions. The normal distribution of powers will not stand in the way of Parliament to pass a law for giving effect to an international obligation even though such law relates to any of the subjects in the state list.
5. In case of failure of constitutional machinery in a state.
Under Art. 356 Parliament is empowered to make laws with respect to all matters in the state list, when the Parliament declares that the government of the state cannot be carried on in accordance with the provisions of the Constitution.

Q. 9 Answer the following questions (in about 20 words each :

(a) What is criminalization of politics?
(b) How is the President of India elected?
(c) What is casting vote?
(d) What is the difference between Council of Ministers and Cabinet?
(e) What is the importance of Right to Constitutional Remedies?
Ans. (a) Nexus between criminals and politicians has paid them rich dividend. Criminals help politicians in winning the election; now-a-day they themselves are contestiong and winning election by might and money.
(b) President of India is elected by an electoral college consisting of elected members of Union Parliament and State Vidhan Sabhas.
(c) Art. 100 provides that the Speaker shall not vote in the first instance, but in case of equality of votes over a motion the speaker is entitled to cast his vote.
(d) Art. 74. (1) states that there shall be a Council of Ministers headed by the Prime Ministers there are different ranks like Cabinet Ministers there are different ranks like Cabinet Minister, independent charge, State and Deputy Minister. Hence council of minister is total number of ministers while cabinet is one of the ranks of it,
(e) Dr. Ambedkar rightly said ‘It is the very soul of the Constitution and the very heart of it. It provides right to move the Supreme Court by appropriate proceedings for the enforcement of the rights conferred by Part III of the Constitution.

Q. 10 Answer any one of the following questions (in about 250 words :

(a) What were the main recommendations of the Platform for Action (PFA) adopted at the Beijing Women Conference, 1995?
(b) Discuss the steps to get ride of child labour in India.

Ans. (a) The fourth World Conference on woman held in Beijing in September 1995 culminated with the adoption of the Beijing Declaration and Platform for Action. The Declaration secured the commitment of government to work towards implementing before the end of the 20th century the strategies agreed on in Naairobi in 1985 and to mobilize resources to achieve the goals set up by the platform.
It incorporates the accomplishments of Prior conferences and treaties such as the universal declaration of Human rights, Convention on the Elimination of all forms of discriminations against woman and the Vienna Declaration. It also reaffirms the definitions set up in Cairo and adds a paragraphs on Human rights in general
The Beijing Conference included agreements aimed at elimination discrimination against women, eradicating poverty and adopting measures towards placing a decisive number of women in key positions. It also recognized that the right of woman to control their security and reproduction is one of their human rights, and called on states to review laws containing punitive measures against women who have undergone abortions. It further posed the need to strengthen legislation protecting the rights of woman.
The summit focused on the following critical areas of concern :
· Inadequacies and inequalities in and unequal access to education and training.
· Inequalities and inadequacies in and unequal access to health care and related services.
· Violence against women.
· The effects of armed or other kinds of conflicts on woman including those living under foreign occupations.
· Inequality in economic structures and policies, in all forms of productive activities and in access to resources.
· Inequality between men and women in the sharing of power and decision making at all levels.
· Insufficient mechanism at all levels to promote advancement of woman.
· Lack of respect for and inadequate promotion and protection of the human rights of women.
· Stereo typing of women and inequality in women’s access to and participation in all communication system especially in the media.
· Gender inequalities in the management of natural resources and in the safeguarding of the environment.
· Persistent discrimination against, and violation of the rights of the girl child.
Hence, we can conclude that agenda at the summit was on gender, equity, equality and empowerment.

(b) Child labour- As India is marching from developing country status to developed country status, there are certain areas of concern like literacy, per capital income, social justice, child labour. Among them child labour is a critical issue. Child labour implies those who are in the age group below 14 years and who work for themselves or their family for an income and who contribute a significant share to the labour force of India.
Children come to work or parent send them to work due to Poverty and this is exploited by the employers. These employers do not even think that the health and growth of a child is being hampered as they employ them. Employers believe in cheap labour. The management of organized and on organized sector do not owe moral or social responsibility.
Apart from poverty illiteracy is also major factor, because parents of such children are not aware of the conquences of child labour to the child labour to the child, to their family and to the society at large.
Unplanned family is also one of the reasons for the parents to send their children to work. They believe in more hands, more income.
Our Constitution Art. 24 and Art. 39 clearly mention that government in all spirit intends to abolish child labour. The Supreme Court in M.C. Mehta Vs State of T.N. 1997 enshrines guidelines how the state authorities should protect economic social and humanitarian rights of million of children working illegally in public and private sectors.
In Aug. 1987 through Comprehensive National policy on child labour, through action plan National child labour projects were launched in 12 child labour endemic districts. Here govt. set up schools to provide non formal/ formal education, vocational training supplementary nutrition, monthly stipend and regular health check ups. This was done to prevent poor child from engaging in employment.
Government has also taken a number of steps against child labour like 12th June celebrated as world day against child labour. National child labour projects scheme provides.
· Special schools/Rehabilitation Centres
· Non formal/formal education
· Mid day meal (It should be strengthened and properly implemented)
· Stipend @ Rs. 100 per child per month also requires review.
· Health care facilities.
The NCLP scheme at present covers 250 districts of India.
The Law and regulations to prevent child labour should be made stringent and with sincere efforts, they should be implemented like child labour (Prohibition and Regulation ) Act. Legislations must be encated in such a way that not a single citizen of India should think of neglecting the provisions in the legislation.
Corporates houses and NGOs along with Govt. agencies should create awareness about the evils of child labour. The children should be made aware of their rights and privileges and when forced to work, the avenues to get rid of it.
Individual campaigning, group campaigning, family campaigning may be resorted to educate parents, children and the promoters of business houses.
Q.11 Amswer any two of the following questions (in about 125 words each) :
(a) What is stealth technology?
(b) Differentiate between National and Cultural heritage.
(c) What is value based politics?
Ans. (a) It is also known as low observability technology and is a sub discipline of electronic counter measures which cover a range of techniques used with aircraft, ships and missiles in order to make them lass visible to radar, infrared and other detection methods. Stealth technology is based on the principle of reflection and absorption that makes aircraft stealthy.
It is a combination of technologies that attempt to greatly reduce the distance at which a vehicle can be detected in particular radar cross section reduction but also acoustic, thermal and other aspects also. The term ‘Stealth’ in reference to reduced radar signature aircraft became popular during the late eighties when f-117 Stealth fighter became widely known. Most stealth aircrafts use matte paint and dark colours and operate only at night. Day light stealth US air force uses grey paints in disruptive schemes : B-2 bomber US aircraft uses chloro fluoro sulphonic acid for evasion. Stealth technology now-a-days is used both in aircrafts like F117 nighthawk, F-22 Raptor, B-2 Sprit Bomber and ship’s like La Fayette, Shivalik Hamina patrol boat.
Stealth technology allows an aircraft to be partially invisible to Rader or any other means of detection. Its does not make aircraft invisible on radar.
(b) The literary meaning of heritages is the history, tradition and qualities that a country or society has had for many years and that are considered an important part of its character.
Now-a-day both the above terms are associated with tourism promotion. The natural heritage mainly consists of physical and biological formations or group of such formation which are outstanding universal value from the aesthetic or scientific point of view like Mt. Everest, Angel fall Geological and physiographical formations and precisely delineated areas which constitute the habitat of threatened species of animals and plants like bio reserves; National parks. Natural sites or precisely delineated natural areas of outstanding universal value from the point of view of science, conservation or natural beauty like Himalays or Sunderban delta.
Cultural heritage on other hand includes monuments, architectural works, sculpture and painting elements of structures of an archaeological nature, inscriptions, cave dwellings and combination of features, which are outstanding universal values like Taj Mahal, Ajanta caves, group of buildings or seprate structure’s made by man known for their architectural and landscape features or unique e.g., Great wall of China, Indus, Egypt sites.
(c) Mahatma Gandhi cited the major structural and collective sins in contemporary period-politics without principles as sin. Hence we can sum up value based politics as based on certain basic principles or ideology which political parties are required to follow. Hence we can say values are equivalent to ideology. An ideology can be defined as a set of values beliefs and principles that guide a political party. All the policies and actions of that political party should be in conformity with these core values. In a democratic system, a political party must also have a share in decision making and hence it enters into electoral politics. But the dilemma that all parties face is, that they seek votes only from those believing in their ideology. As it is not possible because core supporters of any political party are always in limited number, it seeks to broaden its appeal, henceforth compromises and thus creates alliances. Here values are sometimes overlooked or forgotten in order to remain in power. Indian democracy was never in such a state of vaccum so far as the value orientation is concerned. It need value based politics much more than any time in its history. Pseudo values are adopted by political parties which can cause regional differences, caste and religious difference. Now-a-day in India personality based parties cam into existence, which function as private limited companies. Looming danger of free trade economy, environmental degradation, and terrorism require concrete value based politics to safeguard our country from vested interests.
Q.12 Write notes on any two of the following (in about 125 words each) :
(a) Integrated Child Development Services (ICDS)
(b) Prime Minister’s 5-point agenda for India’s development as a knowledge society
(c) The Lokpal Bill
Ans. (a) Integrated Child Development Services ICDS Government of India adopted National Policy for children in Aug. 1974, Under it ICDS was started in 1975 in 33 projects on experimental basis. It was a centrally sponsored scheme, adopting a multi sectoral approach to child development. The objective of ICDS are :
(i) Improving nutrition and health status of children
(ii) Laying foundation for proper psychological and social development of child.
(iii) Reducing mortality, morbidity and school drops outs.
(iv) Achieving effective coordination of policy and its implementation among various departments to promote holistic child development.
(v) Enhancing capability of mother to look after normal health and nutritional needs of child through proper health and nutritional education.
The Strategy adopted in ICDS in the delivery of integrated package of services namely : (i) Supplementary nutrition (ii) Immunization (iii) health check up and reformed services (iv) non formal pre-school education to children (3-6 age) (v) nutrition and health education to women.
Under this programme target group is children upto 6 years; pregnant and lactating mothers. ICDS focuses on rural and tribal areas and backward urban slusm.
(b) Prime Minister’s 5 point agenda for India’s development as a knowledge society.
· To generate cohesiveness among government organizations, industry and educated community to make effective law and their implementation.
· Use of world wide networking and telecommunication to make India knowledge based society.
· Expansion of educational sphere both in urban and rural areas and transform Indian society as enlightened and educated.
· Fuller use and implementation of information technology, telecommunication, science and technology, medical and health science and economic services.
· To generate an allied relationship among the power based technology, economic and information related issues so as to make India a developed nation.
(c) The Lokpal Bill- The Lokpal Bill has a long and chequred history. An agency like the Lokpal would help forces working for a cleaner government. The most recent bill introduced in Parliamnet is the Lok Sabha Bill 2001. It is synonymous to the institution of ombudsman existing in Sweden (Scandinavian countries)
· Its objective was to provide speedy, cheaper form of justice to people
· There member body with a chairperson who is or has been a Chief Justice of Supreme Court and other two members who are or have been judges or C.J. of High Courts.
· The Chairman and other two members shall be appointed by the President on the recommendation of committee consisting of (a) The Vice-President (b) Prime Minister (c) The Speaker (d) Home Minister (e) Leader of house other than the house in which P.M. is a member (f) Leader of the opposition of both the houses.
· In order to make Lokpal office independent following steps have been taken-
(i) Appointment is to be made on the recommendation of committee.
(ii) The Lokpal is ineligible to hold any office of profit under the Government of India or any state or similar such post after retirement.
(iii) Fixed tenure of three years and can be removed only on the ground of proven misbehavior or incapacity after an enquiry made by C.J. and two senior most Judges. Of S.C.
· Lokpal will have its own administrative machinery for conducting investigations.
· Salary of Lokpal is to be charged on the consolidated Fund of India
Jurisdiction of Lokpal
· The central level political functionaries like the council of ministers including the P.M. in relation to letters functions of national security and public order.
· Complaints of offence committed within 10 years from the date of complaint can be taken up for investigation.

Lokpal is supposed to complete the inquiry within a period of six months. The Lok-pal has the The power of civil court to summon any person or authority.
He annually submits its report to the President, which will be put before the both houses of Parliament.
Q. 13 Write short notes on the following (in about 20 words each):
(a) Yakshagana
(b) PACE
(c) Fotloose Industries
(d) The Statue of Liberty
(e) Genome
Ans. (a) Dance-drama of Karnataka, literally means songs of Yaksha, musician known as Himmela and dance group Mummela Product of Vaishnava Bhakti movement based on themes from Puranas, Mahabharata.
(b) it is an acronym of Processor for Aerodynamic Computations and evolutions for super computer developed by “D.R.D.O.s ANURAG”. It is eight gigaflops and target to make it 30 gigaflops.
(c) Industry that can be sited in any of a number of places, often because transport costs are unimportant, having spatially fixed costs which means that the cost of the products do not change despite, where the product is assembled e.g. Diamond and computer chips industries.
(d) Presented to the USA by France in 1886 copper patina clad statue, commemorates the centennial sculptured it. Gustave Effiel and Violet-le-Due were involved in its making.
(e) Coined by Hans Winkler, in 1920, Germany. Its whole hereditary information includes both the genes and the non coding sequences of the DNA.